2017 Flashcards

1
Q

According to Sepsis 3:

a. How do they define organ dysfunction (1)
b. What is needed to define septic shock (2)
c. What are the qSOFA criteria (3)

A

a) life-threatening organ dysfunction caused by a dysregulated host response to infection
* organ dysfunction defined as increase in SOFA score of 2 or more points. Importantly, the SOFA score is an organ dysfunction score. It is not diagnostic of sepsis nor does it identify those whose organ dysfunction is truly due to infection but rather helps identify patients who potentially have a high risk of dying from infection.
b) patients who fulfill the criteria for sepsis (see above) who, despite adequate fluid resuscitation, require vasopressors to maintain a mean arterial pressure (MAP) ≥65 mmHg AND have a lactate >2 mmol/L

c)

  • hypotension (SBP
  • altered mental status (GCS <15)
  • tachypnea (RR>/= 22)

On balance, qSOFA may not be as robust as originally thought and clinicians need to keep in mind that it was originally designed not as a diagnostic tool but rather as a predictive tool that calculates the risk of death from sepsis.

How well did you know this?
1
Not at all
2
3
4
5
Perfectly
2
Q

Patient with severe aortic stenosis goes to operating room for AVR. Has her valve done and post-pump TEE shows normal LV function with some LV hypertrophy. Valve gradients are normal and the valve is functioning well. She is bradycardic and requires pacing. In the ICU she becomes hypotensive and after 1L fluid has not recovered. You suspect she has dynamic LVOT obstruction. They give you a table of values that basically show her HR was paced at 90, and is still paced at 90. Cardiac index went from 3 ~1.6. PAWP went from 10-15, and BP was 120/80 now it is 80/60. They then ask:

a. What are 4 physiologic reasons why her LVOT obstruction is getting worse
b. What are 4 interventions you can perform to improve her hemodynamics

A

a)

  • hypovolemia decreasing preload and therefore LVESV
  • maybe decreased LVESV from arrhythmia, ie. that it’s paced and therefore doesn’t have atrial kick*** Mo agreed and said things that specific to operation
  • low SVR from pump
    • narrow LVOT diameter from hypertrophic LV septum
  • abnormal length or position of mitral valve anterior leaflet
  • hyperdynamic state which increases flow through LVOT thereby worsening Bernoulli effect on septum and mitral valve anterior leaflet
  • post-bypass pump vasoplegia
  • relative tachycardia shortening diastolic filling time
  • ?mismatch between valve

b)

  • decrease HR of pacemaker (this is a guess: try A-V synchronized pacing??? would need to program to VAO)
  • increase preload with fluid bolus
  • increase afterload without inotropy (i.e. phenylephrine or vasopressin)
  • ???early beta blockers
How well did you know this?
1
Not at all
2
3
4
5
Perfectly
3
Q

Patient with diarrhea sees her GP and is found to have thrombocytopenia. He diagnoses ITP. She then gets worse and goes to the ED where they diagnose DIC. She seizes in the ED, CT head is normal. She is hemodynamically stable.

a. List 4 ways to differentiate DIC from ITP
b. What other diagnosis needs to be ruled out in this case?

A

a)

DIC

  • INR prolonged
  • PTT prolonged
  • pt typically looks sick and there should be a precipitant (infection, cancer, trauma, obstetrical issue, etc)
  • fibrinogen is low
  • D-dimer is high
  • blood smear may show schistocytes but this should prompt further investigation for TTP

ITP

  • INR and PTT normal
  • fibrinogen is normal
  • D-dimer is normal
  • red cells look normal on smear

b) TTP

  • pts look sick
  • schistocytes on smear
  • normal INR and PTT (or slight increase)
  • normal fibrinogen
  • normal D-dimer (or slight increase)
How well did you know this?
1
Not at all
2
3
4
5
Perfectly
4
Q

What are the four criteria for major burns?

A

not sure which list they are asking for…

from uptodate a severe burn is one that is:

  • complicated by major trauma
  • inhalational injury
  • chemical burn
  • high voltage electrical burn
  • any burn encompassing >20% of TBSA excluding superficial burns

Burn center referral criteria

  • Partial-thickness burns greater than 10% of TBSA
  • Burns that involve the face, hands, feet, genitalia, perineum, or major joints
  • Third-degree burns in any age group
  • Electrical burns, including lightning injury
  • Chemical burns
  • Inhalation injury
  • Burn injury in patients with preexisting medical disorders that could complicate management, prolong recovery, or affect mortality
  • Any patient with burns and concomitant trauma (such as fractures) in which the burn injury poses the greatest risk for morbidity or mortality. In such cases, if the trauma poses the greater immediate risk, the patient may be stabilized initially in a trauma center before being transferred to a burn unit. Physician judgment will be necessary in such situations and should be in concert with the regional medical control plan and triage protocols.
  • Burned children in hospitals without qualified personnel or equipment for the care of children
  • Burn injury in patients who will require special social, emotional, or rehabilitative intervention
How well did you know this?
1
Not at all
2
3
4
5
Perfectly
5
Q

Obese man with BMI >45, has known ACA aneurysm that is due to be coiled soon (but hasn’t been done yet), has limited mouth opening. What three factors will affect your airway management plan (worded slightly differently).

A

difficulty with BVM

  • obese

difficulty with laryngoscopy

  • limited mouth opening

hemodynamic control

  • avoid excessive hypertension/cough due to unprotected aneurysm
How well did you know this?
1
Not at all
2
3
4
5
Perfectly
6
Q

List the formulas for the following:

a. O2 delivery equation
b. Oxygen extraction ratio
c. Shunt
d. Oxygenation index

A

a) oxygen delivery equation

DO2 = cardiac output X arterial O2 content

= HR X SV X [(1.34 X Hgb concentration X SaO2) + (0.0031 X PaO2)]

b) oxygen extraction ratio

O2 extraction ratio =

(arterial O2 content - venous O2 content) / arterial O2 content

* use arterial O2 content from above but use the following for venous O2 content

venous O2 content = (1.34 X Hgb concentration X SvO2) + (0.0031 X PvO2)

c) shunt

Q shunt / Q total cardiac output

= (pulmonary end-capillary O2 content - arterial O2 content) /

(pulmorary end-capillary O2 content - venous O2 content)

  • pulmorary end-capillary O2 content is estimated from the PAO2
  • arterial O2 content and venous O2 content are calculated from the arterial and mixed venous gas measurements

d) oxygenation index

OI = [mean airway pressure X FiO2 / PaO2] X 100

  • a high OI (eg, ≥25) indicates severe hypoxemic respiratory failure
  • OI is most commonly used in neonates with persistent pulmonary hypertension of the newborn to determine the severity of hypoxemia and to guide the timing of interventions
How well did you know this?
1
Not at all
2
3
4
5
Perfectly
7
Q

Liver transplant patient post-op day 1. Hemodynamically stable but INR and AST/ALT are increasing:

a. What are 4 reasons why the LFT’s are increasing?
b. What one test will you order?

A

a)

  • hepatic artery thrombosis
  • primary graft nonfunction—preservation injury
  • biliary complication
  • sepsis/septicemia
  • too early for cyclosporine (or other drug) toxicity

b) liver ultrasound with doppler do this first…. (vs ?liver biopsy)

The differential diagnosis of liver transplantation dysfunction depends in part upon the time at which it occurs. Within the first few days after transplantation, abnormal liver tests may reflect technical or functional problems, such as hepatic artery thrombosis, preservation injury, biliary anastomosis leakage or stenosis, primary graft nonfunction, or even consequences of shock and septicemia. Although many of these problems may be evident clinically or on imaging such as Doppler ultrasonography, the biopsy findings may sometimes be the first sign of a problem.

We routinely obtain both a Doppler ultrasound and a liver biopsy on the same day when a transplantation recipient presents with hepatic dysfunction.

Acute cellular rejection is generally suspected based upon the development of hepatic biochemical test abnormalities, which may include elevations of some or all of the following: serum aminotransferases, alkaline phosphatase, gamma-glutamyl transpeptidase (GGT), and bilirubin levels. However, these abnormalities are neither sensitive nor specific for distinguishing acute cellular rejection from other causes of hepatic allograft dysfunction and do not correlate with the severity of the rejection episode.

Abnormalities in the hepatic biochemical tests in the short-term post-transplantation period are frequently a manifestation of preservation injury or acute cellular rejection. Preservation injury usually manifests with elevation of the alkaline phosphatase and GGT, without a significant increase in total bilirubin. By contrast, elevation of transaminases (alanine aminotransferase [ALT] and aspartate aminotransferase [AST]) together with rising bilirubin level and/or GGT should raise concern for acute cellular rejection.

Liver histology remains the gold standard for the diagnosis of acute cellular rejection.

How well did you know this?
1
Not at all
2
3
4
5
Perfectly
8
Q

a. What are 3 risk factors for MDR VAP?
b. What are 6 prevention strategies (previous years they have only asked for 4,
they wanted us to list 6 here)?

A

a)

Risk Factors for MDR VAP

  • prior IV Abx use within 90d*
  • septic shock at time of VAP
  • ARDS preceding VAP
  • >/= 5d of hospitalization prior to occurence of VAP
  • acute RRT prior to VAP onset

*(also a risk factor for MDR HAP, MRSA HAP/VAP, and MDR Pseudomonas VAP/HAP)

b)

VAP prevention (specific guideline, see table 2 pg 920):

  • use NIPPV (minimize invasive MV)
  • manage pts without sedation whenever possible
  • interrupt sedation daily
  • assess readiness to extubate daily
  • perform SBTs with sedatives turned off
  • facilitate early mobility
  • utilize ETT with supraglottic suction for pts expected to require >48-72hrs of MV
  • change vent circuit ONLY if visibly soiled or malfunctioning
  • elevate head of bed 30-45degrees

evidence that intervention improves outcomes but insufficient data on possible risks:

  • selective oral/digestive decontamination
  • ???regular oral care with chlorhexidine

no impact on VAP rates/average duration of MV/LoS/mortality:

  • stress ulcer prophylaxis
  • early tracheostomy
  • monitoring gastric residuals
  • early parenteral nutrition
How well did you know this?
1
Not at all
2
3
4
5
Perfectly
9
Q

List two benefits of small bowel over gastric feeding tubes.

A

may reduce the risk of gastroesophageal reflux and pneumonia.

increaes feedomg tolerance

Canadian guideline recommendation

2015 Recommendation:

Based on 16 level 2 studies, small bowel feeding compared to gastric feeding may be associated with a reduction in pneumonia in critically ill patients. In units where small bowel access is feasible, we recommend the routine use of small bowel feedings. In units where obtaining access involves more logistical difficulties, small bowel feedings should be considered for patients at high risk for intolerance to EN (on inotropes, continuous infusion of sedatives, or paralytic agents, or patients with high nasogastric drainage) or at high risk for regurgitation and aspiration (nursed in supine position). Finally, where obtaining small bowel access is not feasible (no access to fluoroscopy or endoscopy and blind techniques not reliable), small bowel feedings should be considered

for those

select patients that repeatedly demonstrate high gastric residuals and are not tolerating adequate amounts of EN intragastrically.

uptodate:

Post-pyloric feeding has theoretical advantages in critically ill patients. Impaired gastric emptying is relatively common and thus feeding beyond the pylorus has the potential to deliver adequate nutrition without the need for parenteral nutrition. In addition, compared to gastric feeding, it may reduce the risks associated with high gastric residuals such as aspiration pneumonia. The delivery of a continuous feed to the jejunum also prevents gastric distension, thus potentially allowing for better respiratory function [30]. The main disadvantages are the inconvenience, risks, and costs associated with placement of the tube beyond the pylorus.

A 2015 meta-analysis included 14 randomized controlled studies that compared gastric versus post-pyloric feeding in 1109 critically ill patients. There were no significant differences in duration of mechanical ventilation or mortality but post-pyloric feeding was associated with a reduction in pneumonia as compared with gastric feeding.

Canadian clinical practice guidelines also noted that small bowel feedings were associated with a lower incidence of pneumonia in critically ill adults (RR 0.77, 95% CI 0.60-1.0), and therefore recommend routine use of small bowel feedings in centers where this is feasible [34]. Canadian ICUs using these guidelines reported higher rates of adequacy of enteral nutrition (percentage of prescribed energy needs actually received) compared with sites less compliant with these guidelines

How well did you know this?
1
Not at all
2
3
4
5
Perfectly
10
Q

Crohns patient requiring TPN

a. What are two ways you can calculate his energy expenditure?
b. How many calories will you give from carbohydrates?
c. How much protein will you give?

A

a)

  • Indirect Calorimetry
  • Predictive Equations
  • Simplistic weight-based equations

Differing opinions in the ASPEN vs Canadian Clinical Practice Nutrition Guidelines…

Canadian Clinical Practice Guidelines (2015, this statement not changed from 2013)

2013 Recommendation:

There are insufficient data to make a recommendation on the use of indirect calorimetry vs. predictive equations for determining energy needs for nutrition or to guide when nutrition is to be supplemented in critically ill patients.

2013 Discussion:

It was noted that both the included studies examined the role of Indirect

Calorimetry (IC) vs. Equations in unselected, heterogeneous patients. There are no randomized controlled trials of the use of IC in select patients (prolonged stay, obesity, for example). Given the differences in the methodological design of the 2 studies i.e. Singer (2011) used indirect calorimetry to guide supplementation of enteral nutrition with parenteral nutrition whereas Saffle (1990) compared the effectiveness of indirect calorimetry guided enteral nutrition to enteral nutrition guided by Curreri formula, the committee agreed not to combine the two studies in a meta-analysis. Similarly the assignment of values was also not thought to be meaningful because of the heterogeneous nature of the studies. There was no signal of benefit in clinical outcomes in either study and the committee noted the signal for harm associated with the use of indirect calorimetry in the Singer study (increase in length of stay, pneumonia and overall infections). The committee decided that there was insufficient evidence to put forward a recommendation.

ASPEN

  • *A3a.** We suggest that indirect calorimetry (IC) be used to determine energy requirements, when available and in the absence of variables that affect the accuracy of measurement. [Quality of Evidence: Very Low]
  • *A3b.** Based on expert consensus, in the absence of IC, we suggest that a published predictive equation or a simplistic weight-based equation (25–30 kcal/kg/d) be used to determine energy requirements. (See section Q for obesity recommendations.)

Clinicians should determine energy requirements to establish the goals of nutrition therapy. Energy requirements may be calculated through simplistic formulas (25–30 kcal/kg/d), published predictive equations, or IC. The applicability of IC may be limited at most institutions by availability and cost. Variables in the ICU that affect the timing and accuracy of IC measurements include the presence of air leaks or chest tubes, supplemental oxygen (eg, nasal cannula, bilevel positive airway pressure), ventilator settings (fractional inspiratory oxygen and positive end-expiratory pressure), continuous renal replacement therapy (CRRT), anesthesia, physical therapy, and excessive movement.26 More than 200 predictive equations have been published in the literature, with accuracy rates ranging from 40%–75% when compared with IC, and no single equation emerges as being more accurate in an ICU.27–32 Predictive equations are less accurate in obese and underweight patients. Equations derived from testing hospital patients (Penn State, Ireton-Jones, Swinamer) are no more accurate than equations derived from testing normal volunteers (Harris-Benedict, Mifflin St Jeor). The poor accuracy of predictive equations is related to many nonstatic variables affecting energy expenditure in the critically ill patient, such as weight, medications, treatments, and body temperature. The only advantage of using weight-based equations over other predictive equations is simplicity. However, in critically ill patients following aggressive volume resuscitation or in the presence of edema or anasarca, clinicians should use dry or usual body weight in these equations.

Table 2. Bundle Statements.

  • Assess patients on admission to the intensive care unit (ICU) for nutrition risk, and calculate both energy and protein requirements
  • to determine goals of nutrition therapy.
  • Initiate enteral nutrition (EN) within 24−48 hours following the onset of critical illness and admission to the ICU, and increase to
  • goals over the first week of ICU stay.
  • Take steps as needed to reduce risk of aspiration or improve tolerance to gastric feeding (use prokinetic agent, continuous infusion,
  • chlorhexidine mouthwash, elevate the head of bed, and divert level of feeding in the gastrointestinal tract).
  • Implement enteral feeding protocols with institution-specific strategies to promote delivery of EN.
  • Do not use gastric residual volumes as part of routine care to monitor ICU patients receiving EN.
  • Start parenteral nutrition early when EN is not feasible or sufficient in high-risk or poorly nourished patients.

b)

Protein (MOST IMPORTANT macronutrient) 1.5g/kg/d, ~20-25% total calories

Carbs/Glucose 3g/kg/d, ~50-60% of total calories

Lipids 0.8g/kg/d, ~25-35% of total calories

Bottom Line: you want 1.5g/kg of protein

for every 1g protein you want 0.5g lipid

for every 1g protein you want 2g carbs

*note this is referring to grams not calories

** if concerned about refeeding give 20kcal/kg/d (down from the typical 25), consider 30 if pt hypermetabolic, head injury or in recovery phase. For obese pts with BMI>30, give 22-25kcal/kg/d and use weight as if their BMI was 25

c)

1.5g/kg of protein

short word on intentional underfeeding: hypocaloric enteral nutrition

2015 Recommendation:

Based on 4 level 2 studies, intentional underfeeding of calories (not protein) should be considered in patients at low nutrition-risk. However, this recommendation does not apply to patients at high nutrition risk.

2015 Discussion:

The committee noted that with the inclusion of 3 new trials (Charles 2014, Petros 2014 and Arabi in press) the effect of hypocaloric enteral nutrition on mortality was associated with a trend towards a reduction in ICU and hospital mortality and a reduction in mechanical ventilation. There was no effect on length of stay outcomes. The differences in calories received were 42-50% in the hypocaloric group vs. 72-75% energy needs in the comparison group yet protein delivery was not different. The committee struggled with the signal of benefit with restricting calories in a heterogeneous ICU patient population and a signal of benefit from optimizing caloric delivery in nutritionally high-risk patients. Given this, it was agreed that a weak recommendation be made for the use of hypocaloric nutrition without underfeeding of protein in nutritionally low-risk patients. It was agreed that the need for maintaining protein intake as demonstrated by recent evidence ought to be emphasized.

But you might intentionally underfeed obese and ARDS pts although most ICU pts are “high nutritional risk”. In those intentionally underfed consider using 5–75% of ideal daily intake (20kcal/kg/d is probably adequate)..however in these ptsa keep protein as is!!!

How well did you know this?
1
Not at all
2
3
4
5
Perfectly
11
Q

Young guy ~23M post ARDS extubated after 3 days with muscle cramping. You suspect he may have PRIS:

a. List 4 risk factors for PRIS
b. What 3 laboratory tests will be abnormal (EXCEPT CK)

A

a)

  • high dose (>4mg/kg/hr)
  • prolonged infusion (>48h)
  • young age
  • critical illness
  • high fat and carbohydrate intake
  • inborn errors of mitochondiral fatty acid oxidation
  • concominant catecholamine infusion
  • concominant steroid therapy
  • traumatic brain injury

b)

  • metabolic acidosis (combination of lactate and renal failure)
  • rhabdomyolosis (increased CK and myoglobin) from direct muscle necrosis of both skeletal and cardiac muscle
  • renal failure
  • hypertriglyceridemia
  • hyperkalemia
  • lipemia
How well did you know this?
1
Not at all
2
3
4
5
Perfectly
12
Q

Trauma patient in periphery with head injury, hemo-pneumothorax, splenic laceration, pelvic fracture, has subclavian line in place, intubated. You are going to be transporting this patient to the trauma center (they don’t say by ground or air). List 6 emergencies you need to be prepared for.

A
  • hemorrhagic shock from intra-abdominal or pelvic bleeding
  • obstructive shock from worsening hemo-pneumothorax
  • development of worsened head injury, increased ICP
  • airway issue (are they intubated?)
  • ?spinal shock from ?SCI
  • ?injury from CVC insertion (bleeding or new pneumothorax)
How well did you know this?
1
Not at all
2
3
4
5
Perfectly
13
Q

77F with ischemic bowel who requires laparotomy. She has known CHF with EF 20%, COPD with FEV1 30% predicted, she is on CRRT, and has been trached with prolonged ventilation following her necrotizing pancreatitis. She is not doing well. You have a family meeting and the family thinks that she would want to pursue all active treatment including surgery. List two ethical principles that might suggest this is a bad idea, explain your reasoning.

A

non-maleficence

beneficence (should have net benefit for the pt)

Autonomy (was this discussed with the pt)

  • Requires that the patient have autonomy of thought, intention, and action when making decisions regarding health care procedures. Therefore, the decision-making process must be free of coercion or coaxing. In order for a patient to make a fully informed decision, she/he must understand all risks and benefits of the procedure and the likelihood of success. Because ARTs are highly technical and may involve high emotions, it is difficult to expect patients to be operating under fully-informed consent.

Justice

  • The idea that the burdens and benefits of new or experimental treatments must be distributed equally among all groups in society. Requires that procedures uphold the spirit of existing laws and are fair to all players involved. The health care provider must consider four main areas when evaluating justice: fair distribution of scarce resources, competing needs, rights and obligations, and potential conflicts with established legislation. Reproductive technologies create ethical dilemmas because treatment is not equally available to all people.

Beneficence

  • Requires that the procedure be provided with the intent of doing good for the patient involved. Demands that health care providers develop and maintain skills and knowledge, continually update training, consider individual circumstances of all patients, and strive for net benefit.

Non-maleficence

  • Requires that a procedure does not harm the patient involved or others in society. Infertility specialists operate under the assumption that they are doing no harm or at least minimizing harm by pursuing the greater good. However, because assistive reproductive technologies have limited success rates uncertain overall outcomes, the emotional state of the patient may be impacted negatively. In some cases, it is difficult for doctors to successfully apply the do no harm principle.
How well did you know this?
1
Not at all
2
3
4
5
Perfectly
14
Q

What are two hemodynamic profiles that you might encounter in a patient post cardiac arrest with post cardiac arrest syndrome (or something, I had no idea what they were looking for here).

A

see paper

  • anoxic brain injury
  • arrest-related myocardial dysfunction
  • systemic ischemic-reperfusion response
  • persistent precipitating pathology (ACS, chronic ischemic scar, PR, cardiomyopathies)

so for the hemodynamic profiles, I guess I would say hypotension from distributive shock, or cardiogenic shock due to stunning or primary cardiac problem (CM, PE, ACS)

How well did you know this?
1
Not at all
2
3
4
5
Perfectly
15
Q

Picture of purpuric rash on legs – with stem that reads: 19F university students presents with fever, myalgias, neck stiffness and this rash.

a. What is the causative organism
b. What type of precautions are required

A

a) Neisseria meningitidis
b) droplet and contact until 24hrs after initiation of appropriate antibiotic therapy

Manifestations of Meningococcal infection

Acute systemic meningococcal disease is most frequently manifest by three syndromes:

  • Meningitis
  • Meningitis with accompanying meningococcemia
  • Meningococcemia without clinical evidence of meningitis

The typical initial presentation of meningitis due to N. meningitidis consists of the sudden onset of fever, nausea, vomiting, headache, decreased ability to concentrate, and myalgias in an otherwise healthy patient. The first classic symptom of meningococcal disease was rash, which sometimes evolved from nonspecific to petechial to hemorrhagic over several hours.

The petechial rash appears as discrete lesions 1 to 2 mm in diameter, most frequently on the trunk and lower portions of the body. The mucous membranes of the soft palate, ocular, and palpebral conjunctiva must be carefully examined for signs of hemorrhage. Over 50 percent of patients will have petechiae upon presentation [2,7]. Petechiae can coalesce into larger purpuric and ecchymotic lesions (picture 1). The petechiae correlate with the degree of thrombocytopenia and clinically are important as an indicator of the potential for bleeding complications secondary to disseminated intravascular coagulopathy (DIC).

A maculopapular eruption resembling a wide variety of viral exanthems, particularly rubella, can be an early finding in meningococcemia. This transient rash generally does not persist for more than two days and has frequently disappeared hours after its first observation; it is neither purpuric nor pruritic.

Purpura fulminans is a severe complication of meningococcal disease [16], occurring in approximately 15 to 25 percent of those with meningococcemia [17,18]. It is characterized by the acute onset of cutaneous hemorrhage and necrosis due to vascular thrombosis and disseminated intravascular coagulopathy. Initially, there is cutaneous pain followed by erythema and petechiae. Ecchymoses develop and these lesions, if the course is not altered with therapy, evolve into painful indurated, well-demarcated purple papules with erythematous borders. These areas progress to necrosis with formation of bullae and vesicles. Gangrenous necrosis can follow with extension into the subcutaneous tissue and occasionally involves muscle and bone.

b) Droplet precautions should be continued until 24 hours after institution of effective antibiotics in patients with suspected or confirmed N. meningitidis infection.

Chemoprophylaxis is indicated in close contacts of patients with meningococcal infection and should be given as early as possible following the exposure. Although “close contact” has not been clearly defined, it generally refers to individuals who have had prolonged (>8 hours) contact while in close proximity (<3 feet) to the patient or who have been directly exposed to the patient’s oral secretions during the seven days before the onset of the patient’s symptoms and until 24 hours after initiation of appropriate antibiotic therapy

How well did you know this?
1
Not at all
2
3
4
5
Perfectly
16
Q

Patient with difficult percutaneous tracheostomy. She develops subcutaneous emphysema and CXR shows a small right sided pneumothorax. They give you a Bronchoscopy picture which is blurry and unclear but shows a hole just above the carina.

a. What is the most important next management step?
b. What is shown in the picture?
c. Her volumes begin to drop and subcutaneous emphysema gets worse, what are the next two things you will do?
d. What are 2 absolute and 2 relative contraindications to percutaneous tracheostomy?

A

a)

?confirm appropriate placement of tracheostomy tube by bronch +/- end-tidal CO2

b)

?tracheal injury

c)

?insert right chest tube

?reduce airway pressures

?deepen ETT to try and bypass tracheal injury (bronch-guided)

call thoracic surgery

d)

From uptodate

absolute contraindications (they didn’t list any absolute contraindications)

relative contraindications

  • age <15
  • uncorrectable bleeding diathesis
  • gross distortion of the neck from hematoma, tumour, thyromegaly, scarring
  • tracheomalacia
  • evidence of soft tissue infection of the neck
  • obese/short necks which obscure landmarks
  • inability to extend neck due to cervical fusion, RA, etc.

A study that evaluated the rates of bleeding complications during percutaneous tracheostomy showed that bleeding complications could be predicted by a platelet count less than 50,000, an activated partial thromboplastin time longer than 50 seconds, or the presence of two or more coagulation disorders [79]. Administration of prophylactic subcutaneous heparin did not increase the risk of bleeding.

another resource

absolute:

  • infants
  • infection at insertion site
  • operator inexperience
  • unstable cervical injuries
  • uncontrollable coagulopathy

relative:

  • enlarged thyroid glands
  • presence of pulsatile vessel at insertion site
  • difficult anatomy
  • coagulopathy
  • close proximity to burns/surgical wounds
  • high PEEP/FiO2 requirements
  • history of cervical injury or tracheostomy
  • high riding inominate artery
  • radiotherapy to cervical region in last 4wks
  • controlled local infection

Percutaneous tracheostomy should NOT be performed if EMERGENCY

pt refusal

How well did you know this?
1
Not at all
2
3
4
5
Perfectly
17
Q

They give you a diagram of a transducer levelled to 10cm below a column of water.

a. The transducer is inserted to 10cm below the water column (right where the level is) what will the transducer read?
b. The transducer tip is then inserted to 40cm below the water column. What will the transducer read?

A

a) 0cm H2O
b) +30cm H2O

or +10 H2O and +40 H2O but if it’s levelled at 10 then I suspect this means it’s calibrated to zero at that level

levelling is not necessarily the same as zeroing

How well did you know this?
1
Not at all
2
3
4
5
Perfectly
18
Q

Patient with GBS:

a. List 2 evidence based treatments that will improve clinical course.
b. What respiratory parameters will you look for to suggest they may need to be intubated?

c) another test asked to list 2 physical exam features that may predict respiratory failure/needing intubation.

A

a)

Plasma exchange

IVIG

The main modalities of therapy for Guillain-Barré syndrome (GBS) are plasma exchange (also called plasmapheresis) and administration of intravenous immune globulin (IVIG). These treatments hasten recovery from GBS, as shown in randomized controlled trials. Patients recover sooner when treated early. The beneficial effects of plasma exchange and IVIG are believed to be equivalent, while combining the two treatments is not beneficial.

The choice between plasma exchange and IVIG is dependent on local availability and on patient preference, risk factors, and contraindications. Patients who are mildly affected and already recovering do not require disease-modifying therapy. When both therapies are equally available and there are no contraindications for either, we suggest treatment with IVIG. Intravenous immune globulin is given for five days at 0.4 gram/kg per day. Side effects include aseptic meningitis, rash, acute renal failure (mostly related to sucrose containing products), and (rarely) hyperviscosity leading to stroke. IgA deficiency can lead to anaphylaxis.

Aside from plasma exchange and IVIG, no other pharmacologic agents have been found to be effective for GBS. In particular, glucocorticoids are not beneficial. In a systematic review and meta-analysis of six trials with 587 participants, glucocorticoid-treated patients with GBS showed no significant difference in disability grade compared with patients who were not treated with glucocorticoids.

b)

The respiratory status of patients with Guillain-Barré syndrome (GBS) can deteriorate quickly and unpredictably. Many patients require mechanical ventilation. Respiratory features associated with progression to respiratory failure in patients with severe GBS include:

  • Vital capacity less than 20 mL/kg
  • MIP less than 30 cmH2O
  • MEP less than 40 cmH2O, or a reduction of more than 30 percent in vital capacity

Clinical features associated with respiratory failure include:

  • Time of onset to admission less than seven days
  • Inability to cough
  • Inability to stand
  • Inability to lift the elbows
  • Inability to lift the head

Bulbar dysfunction, autonomic dysfunction, and bilateral facial palsy are also associated with the need for mechanical ventilation.

How well did you know this?
1
Not at all
2
3
4
5
Perfectly
19
Q

Patient with recent infection, ascending paralysis and you suspect GBS:

a. What is the most common infectious cause for this syndrome
b. What are two other infections that cause this system (?syndrome)

A

a)

Campylobacter jejuni infection is the most commonly identified precipitant of GBS. Furthermore, campylobacter-associated GBS appears to have a worse prognosis, manifested by slower recovery and greater residual neurologic disability, than other forms of the disease.

b)

GBS also occurs in association with human immunodeficiency virus (HIV) infection, predominantly in those who are not profoundly immunocompromised.

Multiple reports have found an increased risk of GBS following influenza-like illnesses.

Less commonly, GBS has been reported following infection with varicella-zoster virus, herpes simplex virus, hepatitis A, B, C, and E viruses, and the bacteria Haemophilus influenzae, Escherichia coli, and Mycoplasma pneumoniae. The importance of these infectious agents as triggers of GBS is uncertain.

How well did you know this?
1
Not at all
2
3
4
5
Perfectly
20
Q

Trauma patient with traumatic injuries post-OR for damage control laparotomy, femur fractures bilaterally not fixed, abdomen packed:

a. 180/100, HR 130, Pulsus 20 what ONE management step will you take?
b. BP 120/60 HR 130 pulsus 20 on norepi 0.3mcg/kg/min what management step will you take?
c. BP 120/60 HR 130, pulsus 20, on NE 0.3mcg/kg/min patient has had 5L crystalloid 6U packed red cells and has ongoing transfusion requirements – what ONE management step will you take next?
d. BP 120/60 HR 130 Pulsus 20 on NE 0.3mcg/kg/min has sats of 90% and ECHO shows RV strain. What do you think the diagnosis is.

A

a) pain control
b) fluid/blood resuscitation
c) apply splint and traction to both legs…Mo said pericardial tamponade
d) fat embolism

Fat embolism syndrome (FES) typically manifests 24 to 72 hours after the initial insult, but may rarely occur as early as 12 hours or as late as two weeks after the inciting event [37]. Affected patients develop a classic triad: hypoxemia, neurologic abnormalities, and a petechial rash. None of these features are specific for FES.

Pulmonary manifestations are the most common presenting features of FES. Hypoxemia, dyspnea, and tachypnea are the most frequent early findings. In one series, hypoxemia was present in 96 percent of cases [38]. A syndrome indistinguishable from acute respiratory distress syndrome (ARDS) may develop. Approximately one-half of patients with FES caused by long bone fractures develop severe hypoxemia and require mechanical ventilation.

Neurologic abnormalities are also common and typically manifest after respiratory abnormalities, although rare case reports suggest neurological symptoms can occur in isolation [40]. Neurologic manifestations range from the development of an acute confusional state and altered level of consciousness to seizures and focal deficits.

The characteristic red-brown petechial rash may be the last component of the triad to develop and occurs in only 20 to 50 percent (on average one third) of cases (picture 1) [38,39,42]. It is found most often on the nondependent regions of the body including the head, neck, anterior thorax, axillae, and sub-conjunctiva.

The only therapy for fat embolism syndrome is supportive care.

How well did you know this?
1
Not at all
2
3
4
5
Perfectly
21
Q

Patient with burns to 65% BSA, mostly chest abdomen and extremities. He is admitted and you start resuscitation as per parkland formula. 5 hours later his peak pressures are reading 50cm H20 and plateau are 40cmH20. CXR is normal. CVP is 30. What are two causes for this and what is one thing you will do about each?

A
  • abdominal compartment syndrome -> decompression with laparotomy
  • chest wall rigidity/low compliance from circumferential burn -> chest escharotomy
  • …less likely I guess might be pulmonary edema/ARDS for which you might reduce resuscitation and consider RRT/diuresis
How well did you know this?
1
Not at all
2
3
4
5
Perfectly
22
Q

Asthmatic girl intubated and put on RR 32, VC 600mL, PEEP 8, on FiO2 70%, you get called 15min later when BP is 70/30 what is the next step in management?

A

lacks enough detail to clearly answer but her RR and Vt are probably too high leading to dynamic hyperinflation. rule out pneumothorax

Immedicately I would disconnect pt from ventilator and push on chest/abdo then give IVF, reduce RR and Vt.

How well did you know this?
1
Not at all
2
3
4
5
Perfectly
23
Q

Patient with ARDS, intubated and on low tidal volume ventilation of 6mL/kg, PEEP 5, Volumes 360mL, Inspiratory trigger -0.5cmH2O. develops a bronchopleural fistula and has CT inserted to -20cmH2O. He is paralyzed with TOF 0 and no activity. RR set to 15 but persistently has RR 35 with alkalotic ABG and decreased CO2.

a. What is one ventilator change you can make to improve the blood gas?
b. What is one non-ventilator change you can make to improve the blood gas?
c. How is the best way to measure the air leak?
d. What are four ventilator strategies you can use to treat a bronchopleural fistula?

A

a) change to fully controlled mode of ventilation (or increase inspiratory trigger)
b) decrease chest tube suction pressure (i.e. less negative)

c)

A BPF can be quantified from ventilator graphics as volume that is “lost” – that is, the difference between the inspiratory and expiratory tidal volumes (see images)

d)

decrease mean airway pressure, decrease tidal volume more specific

decrease intrinsic PEEP

maximize negative pressure or spontaneous ventilation

extubate as early as possible

When any type of pulmonary barotrauma is detected, immediate attempts should be made at the bedside to lower the plateau airway pressure.

  1. minimize tidal volume and PEEP
  2. minimize chest tube suction
  3. use negative pressure (i.e. spontaneous breathing) rather than positive pressure ventilation (or minimize positive pressure ventilated breaths)

Guidelines for ventilator management in the patient with airleak from barotrauma and alveolopleural fistula

  • Reduce both mean airway pressure and the number of positive-pressure breaths, using the lowest number of mechanical breaths that permits acceptable alveolar ventilation
    • Wean patient completely if possible
    • Partial ventilatory support (eg, low-rate SIMV or pressure support) is preferable to total ventilatory support (eg, assist/control, high-rate SIMV, or pressure control ventilation)
    • Avoid or correct respiratory alkalosis (to minimize minute ventilation)
    • Consider use of permissive hypercapnia (reducing minute ventilation and allowing arterial PCO2 to rise)
  • Limit effective (returned) tidal volume to 5 to 8 mL/kg
  • Minimize inspiratory time, and hence mean airway pressure:
    • Keep inspiration-to-expiration ratio low (eg, 0.33)
    • Use high inspiratory flow rate (eg, 70 to 100 L/min)
    • Avoid inflation hold (end-inspiratory pause) and inverse-ratio ventilation
    • Use low-compressible-volume (non-disposable) ventilator circuit
  • Minimize PEEP (both dialed-in and auto-PEEP)
  • Use least amount of chest tube suction that maintains lung inflation
  • If spontaneous movement exacerbates leak, keep patient heavily sedated (in unusual circumstances neuromuscular blockade may also be necessary)
  • Explore position differences, and avoid patient positions that increase the leak
  • Treat bronchospasm and other causes of expiratory airflow obstruction
  • Consider specific or unconventional measures (eg, independent lung ventilation, high-frequency jet ventilation, PEEP to chest tubes, etc) only if the air leak per se is clinically felt to be worsening the patient’s condition (eg, intractable hypotension or arrhythmias in association with respiratory acidosis)
  • Treat underlying cause of respiratory failure, maintaining nutritional and other support, with goal of discontinuing mechanical ventilation as soon as possible
How well did you know this?
1
Not at all
2
3
4
5
Perfectly
24
Q

Patient post mitral valve and aortic valve replacement with cardiogenic shock post bypass on high doses of milrinone, epi, norepi. Has IABP inserted. You see blood in the gas line:

a. What is the problem?
b. What is your next step in management?

A

a) possible rupture of IABP balloon
b) ?early removal of IABP (before it thromboses) or call CV surgery and have them assess it (may need to be removed in OR)

Blood in the inflation catheter could imply rupture of the balloon. It needs to be notified immediately to the cardiothoracic surgical team. If the situation is not promptly attended to and clots form in the balloon it will not be able to be removed without causing femoral artery injury. The blood rapidly reacts with the helium causing a hard clot formation, which together with the tortuous atherosclerotic aortic environment results in entrapment of a semi-deflated balloon.

Balloon rupture and entrapment is a rare complication in patients treated with an IABP, and it is often difficult to detect. Management should be aimed at early rupture detection with a low threshold to remove the IABP before entrapment can occur. In case of IABP entrapment, either percutaneous removal in conjunction with local or systemic thrombolysis or surgical extraction should be performed, followed by repair of the damaged iliofemoral arteries.

How well did you know this?
1
Not at all
2
3
4
5
Perfectly
25
Q

a) What four electrolyte abnormalities do you see in tumor lysis syndrome?
b) What two drugs do you give to prevent/treat tumor lysis?

A

a)

hyperkalemia

hyperuricemia (from catabolism of nucleic acids)

hyperphosphatemia (and secondary hypocalcemia)

b) would these count as two drugs???

  • rasburicase or allopurinol
  • IV fluids

The main prophylactic strategies are intravenous (IV) hydration and the use of hypouricemic agents, such as allopurinol and rasburicase.

IV hydration — Aggressive IV hydration is the cornerstone of preventing TLS and is recommended prior to therapy in all patients at intermediate or high risk for TLS (table 3) [2]. The goal of IV hydration is to improve renal perfusion and glomerular filtration, and induce a high urine output to minimize the likelihood of uric acid or calcium phosphate precipitation in the tubules. A 2008 International Expert Panel on TLS recommended that both children and adults at risk for TLS initially receive 2 to 3 L/m2 per day of IV fluid.

Urinary alkalinization — The role of urinary alkalinization with either acetazolamide and/or sodium bicarbonate is unclear and controversial. In the past, alkalinization to a urine pH of 6.5 to 7 or even higher was recommended to increase uric acid solubility, thereby diminishing the likelihood of uric acid precipitation in the tubules. However, this approach has fallen out of favor.

Allopurinol can be used in intermediate risk pts, and rasburicase in high risk pts as a preventive treatment of TLS.

Treatment of established TLS

Patients who present with or develop TLS during therapy should receive intensive supportive care with continuous cardiac monitoring and measurement of electrolytes, creatinine, and uric acid every four to six hours [50]. Effective management of these cases involves the combination of treating specific electrolyte abnormalities, the use of rasburicase at 0.2 mg/kg (if it was not given initially) with repeated doses as necessary, attempting to wash out the obstructing uric acid crystals with fluids with or without a loop diuretic, and the appropriate use of renal replacement therapy.

Symptomatic hypocalcemia should be treated with calcium at the lowest doses required to relieve symptoms. In most situations, clinicians should use other oral phosphate binders, even though there are no good studies demonstrating efficacy.

How well did you know this?
1
Not at all
2
3
4
5
Perfectly
26
Q

You are a hospital administrator and you want to access a database containing illness severity data.

a. What are three things you can do with that information?
b. What is one measurement tool to assess staff workload on your unit (i.e. nursing workload)?
c. APACHE and SOFA are two scoring systems used to assess illness severity. What patient population is NOT included in the development of these scoring systems?

A

a)

predict hospital mortality

predict ICU length of stay

???resource allocation

b) nursing activities score

therapeutic intervention scoring system-28 (TISS-28)

c) ???…ahh couldn’t find this after looking for sooo long but I might guess burn pts

“In addition, in most of the scores, specific populations were excluded from the original databases (for example, patients with burns, patients aged less than 16 or 18 years, patients with a very short length of ICU stay, and so on).”

Acute Physiologic and Chronic Health Evaluation (APACHE) — The APACHE scoring system is widely used in the United States, of which there are four versions (APACHE I through IV) [4-11]. APACHE instruments require the input of a large number of clinical variables, from which a severity score is derived. The resulting severity score is entered into a computer-generated logistical regression equation, which predicts hospital mortality and in some cases length of stay. The required variables differ among the versions but generally include factors such as age, diagnosis, prior treatment location, and numerous acute physiologic and chronic health variables.

Sequential (sepsis-related) Organ Failure Assessment (SOFA) — The SOFA score was initially designed to sequentially assess the severity of organ dysfunction in patients who were critically ill from sepsis. The original SOFA instrument was derived from a cohort of 1449 patients admitted to 40 ICUs in 16 countries [28]. Since multiple organ dysfunction is common in critically ill patients, it has since been used to predict mortality in those with organ failure from other causes including those with acute liver failure from acetaminophen overdose, chronic liver failure (CLIF-SOFA), and cancer, as well as in patients who have undergone cardiac surgery or hematopoietic stem cell transplant [29-34].

SOFA uses simple measurements of major organ function to calculate a severity score (calculator 3). The scores are calculated 24 hours after admission to the ICU and every 48 hours thereafter (thus, the term “Sequential” Organ Failure Assessment). The mean and the highest scores are most predictive of mortality. In addition, scores that increase by about 30 percent are associated with a mortality of at least 50 percent

How well did you know this?
1
Not at all
2
3
4
5
Perfectly
27
Q

a) What are the four criteria to be an organ donor after cardiac death (DCD)?
b) What are two recommendations for declaration of death in DCD?

A

Had trouble with both of these questions…???

a)

Before considering donation, the patient should be judged to have:

  • A non-recoverable injury or illness
  • Dependence on life-sustaining therapy
  • Intention to withdraw life-sustaining therapy, and
  • Anticipation of imminent death after withdrawal of life-sustaining therapy.

As a general rule, eligibility criteria are similar to those for organ donation after NDD and should be based on demographic, age and organ-function criteria detailed in the previous CCDT forum. Patients with a history of intravenous drug abuse, sepsis or serious systemic infection, or active malignancies and high-grade brain tumours are excluded. Bacteremic patients are not necessarily excluded. Patients with non-melanoma skin malignancies and some primary non-metastatic brain tumours may be eligible. Organs contaminated with hepatitis B, C or HIV may be transplanted into recipients already infected with these same viruses. Infections with human T-cell leukemia-lymphoma virus, systemic viral infection (e.g., measles, rabies, adenovirus), prion-related disease and herpetic meningoencephalitis are contraindications for organ donation.

b)

determination of fact of death - must be determined by 2 physicians in accordance with “accepted medical practice”

conflict of interest - no physician who has had any association with a proposed transplant recipient that might influence their judgement shall take any part in the determinatio nof death of the donor

prohibition on participation in transplant - no physician who took any part in the determination of the fact of death of the donor shall participate in any way in transplant procedures

determination of cardiocirculatory death -

  • Beginning with the onset of circulatory arrest, there must be a 5-minute period during which the absence of palpable pulses, blood pressure and respiration are continuously observed by at least 1 physician and
  • Death is determined by 2 physicians by documenting the absence of palpable pulses, blood pressure and respiration on completion of this 5-minute period.

The physician present during the 5-minute period of continuous observation and who makes 1 of the determinations of death must be a staff physician with the requisite skill and training.

How well did you know this?
1
Not at all
2
3
4
5
Perfectly
28
Q

COPD patient passess SBT.

a) What Is one evidence based way to prevent reintubation?
b) What are four risk factors for reintubation in someone who has passed an SBT?

A

a) extubation to NIPPV

b)

  • reduced cough peak expiratory flow (PEF <60L/min) or unable to moisten index card held in front of ETT
  • increased sputum volume (>2.5mL/hr)
  • ???impaired neurologic function (inability to follow commands) ***but controversial and not always shown

results of one study also add:

  • higher illness severity at admission
  • higher amount of secretions
  • higher minute ventilation
  • higher number of failed SBTs
  • lower oxygenation

advanced age

cardiac disease

hypercapneic resp failure

How well did you know this?
1
Not at all
2
3
4
5
Perfectly
29
Q

Patient on CVVHD:

a. What are two ways in increase solute clearance?
b. What two drug characteristics that will affect clearance?

A

a)

  • increase dialysate flow
  • using a low solute bath
  • increase blood flow rate (may need a dialysis and not CRRT filter)
  • change to CVVHDF (would use post filter fluid replacement)
  • use a higher flux membrane (all membranes used for CRRT should be high flux already)

b)

  • degree of protein binding (only unbound drug can be removed by CRRT)
  • volume of distribution
  • and potentially molecular weight
  • water solubility
  • +/- interactions of the drug with dialyzer/hemofilter membrane

As noted above, convective therapies (CVVH) may clear large solutes better than diffusive therapies (CVVHD and CVVHDF). However, for most drugs that are used in the intensive care unit (ICU), the differences in clearance provided by different modalities are negligible.

uptodate section on CVVHD

Continuous venovenous hemodialysis (CVVHD) — In CVVHD, plasma is exposed to drug-free dialysate, which runs countercurrent to plasma flow through the dialyzer. Drug removal during CVVHD occurs primarily by diffusion into the drug-free dialysate. The removal of a solute by dialysis is dependent on the degree to which the solute diffuses across a particular membrane. Convection is less important (though can occur) since the ultrafiltration rate is held at a much lower level than with hemofiltration alone.

In addition to the volume of distribution, protein binding, and drug-membrane interactions, drug removal by CVVHD is influenced by drug molecular weight and the dialysate flow rate [33].

Diffusion rates for any solute are indirectly related to solute molecular mass. Thus, the rate of solute transfer by diffusion decreases as the molecular size increases. This is by contrast to solute transfer by convection, which, for most solutes, is not affected by solute molecular size. As a result, CVVH may theoretically be more effective than CVVHD in removing cytokines and solutes with relatively higher molecular weights.

In practice, however, CVVH does not appear to provide significantly better middle-molecule clearance compared with CVVHD.

30
Q

Delirium is a major issue in the ICU. List 4 non-pharmacologic treatments for delirium.

A
  • cognitive stimulation
  • eye glasses
  • hearing aids
  • reorientation (verbal or with clocks, calendars)
  • early mobility
  • ear plugs, minimize disruptions at night
  • window
  • minimize restraint use

from PADIS guidelines:

Question: Should a multicomponent, nonpharmacologic strategy (vs no such strategy) be used to reduce delirium in critically ill adults?

Recommendation: We suggest using a multicomponent, nonpharmacologic intervention that is focused on (but not limited to) reducing modifiable risk factors for delirium, improving cognition, and optimizing sleep, mobility, hearing, and vision in critically ill adults (conditional recommendation, low quality of evidence).

Remarks: These multicomponent interventions include (but are not limited to) strategies to reduce or shorten delirium (e.g., reorientation, cognitive stimulation, use of clocks); improve sleep (e.g., minimizing light and noise); improve wakefulness (i.e., reduced sedation); reduce immobility (e.g., early rehabilitation/mobilization); and reduce hearing and/or visual impairment (e.g., enable use of devices such as hearing aids or eye glasses).

31
Q

28M with trauma presents to trauma bay with low GCS, has positive FAST and goes directly to the OR found to have splenic lac, post splenectomy. Now hemodynamically stable and returning to ICU. What is the ONE next thing you will do?

A

wake him up for neurologic exam?…secondary survey as per Mo

32
Q

Man sent home from ICU after long stay with prolonged delirium and ventilation and shock. Now at home with wife, needs help to get dressed, can’t focus on the news, and he says he’s always tired.

a. What is the most likely diagnosis?
b. What are four other possible diagnosis?

A

a) post-intensive care syndrome

The clinical presentation of post-intensive care syndrome (PICS) includes a constellation of cognitive, psychiatric, and physical signs and symptoms with the hallmark feature that they are newly-recognized or worsened after a critical illness. Common symptoms include weakness, poor mobility, poor concentration, fatigue, anxiety, and depressed mood, which are corroborated by examination and formal testing. Although recovery is possible, many of the signs and symptoms of PICS last for months to years.

COGNITION - The severity of cognitive impairment varies from mild to severe – from subtle difficulties in accomplishing complex executive tasks to a profound inability to conduct one’s activities of daily living.

The areas of cognition that are commonly affected in PICS include the following:

  • Attention/concentration
  • Memory
  • Mental processing speed
  • Executive function

PSYCHIATRIC IMPAIRMENT - Psychiatric morbidity after critical illness is often disabling, and is associated with reduced quality of life for both the patient and their family. The mood disorders most commonly encountered in patients with PICS include anxiety, depression and posttraumatic stress disorder (PTSD).

PHYSICAL IMPAIRMENT - Patients with PICS may exhibit the signs and symptoms of intensive care unit (ICU)-acquired weakness that ranges from generalized poor mobility and multiple falls to quadriparesis and tetraparesis. These signs and symptoms frequently lead to persistent disabilities in activities of daily living and instrumental activities of daily living (eg, ability to take medications, perform housework).

b) ???

  • ICU-acquired weakness
  • hypothyroidism
  • dementia
  • depression
  • PTSD
  • anxiety
33
Q

List 3 pathophysiologic changes in ARDS.

A

Is this what they are referring to?

pathologic stages:

  1. exudative stage
  2. proliferative stage
  3. fibrotic stage

NEJM article

Exudative stage: This phase is characterized by innate immune cell mediated damage of the alveolar endothelial and epithelial barriers and the accumulation of protein-rich edema fluid within the interstitium and alveolus. Although ARDS can occur in the absence of neutrophils, excessive neutrophilic inflammation is a major contributor to lung injury in ARDS. Activated neutrophils contribute to injury by releasing pre-stored inflammatory mediators, reactive oxygen species (ROS) and proteinases. The resultant injury leads to loss of basement membrane integrity, epithelial-endothelial barrier disruption and culminates in interstitial and intra-alveolar flooding.

Proliferative stage: The repair programs initiated during the second or proliferative phase of ARDS are essential for host survival. The formation of a provisional fibrin/fibronectin-rich matrix along the denuded basement membranes is conducive to the proliferation and accumulation of fibroblasts, myofibroblasts and locally generated pluripotent mesenchymal progenitor cells to allow for repair.

Fibrotic stage: The final fibrotic phase of ARDS does not occur in all patients but has been linked to prolonged mechanical ventilation and increased mortality. Even in the low-tidal volume era, ventilator-induced lung injury may continue to be a major contributor to the development of lung fibrosis.

34
Q

a) List 4 medications besides benzos to treat status epilepticus.
b) What are two ion channels and list what the drugs do (i.e. open, close, block)?Do not list the name of the drug.
c) What does the GABA agonist do to the action potential of the cell?

A

a) phenytoin: Stabilizes neuronal membranes and decreases seizure activity by increasing efflux or decreasing influx of sodium ions across cell membranes in the motor cortex during generation of nerve impulses.

valproic acid: Intravenous valproic acid is increasingly used in the treatment of status epilepticus. It is preferred over phenytoin in patients with primary generalized epilepsies, although these patients represent a relatively small proportion of those with GCSE. Causes increased availability of gamma-aminobutyric acid (GABA), an inhibitory neurotransmitter, to brain neurons or may enhance the action of GABA or mimic its action at postsynaptic receptor sites. Also blocks voltage-dependent sodium channels, which results in suppression of high-frequency repetitive neuronal firing.

levetiracetam: The precise mechanism by which levetiracetam exerts its antiepileptic effect is unknown. However, several studies have suggested the mechanism may involve one or more of the following central pharmacologic effects: inhibition of voltage-dependent N-type calcium channels; facilitation of GABA-ergic inhibitory transmission through displacement of negative modulators; reduction of delayed rectifier potassium current; and/or binding to synaptic proteins which modulate neurotransmitter release.

phenobarbital: Phenobarbital is a very effective anticonvulsant, especially in the acute management of seizures, but it was not the best initial treatment in the VA comparative trial [3]. Various studies have shown a rate of seizure control of approximately 60 percent when phenobarbital is used alone, similar to that with lorazepam alone or the combination of phenytoin and diazepam [3,60]. High doses of phenobarbital will control almost any seizure [61], but at the cost of substantial sedation and potential reduction of blood pressure and respiration. Despite its efficacy, phenobarbital is generally not used as a first-line treatment in adults because administration is slow, it causes prolonged sedation, and it may involve a higher risk of hypoventilation and hypotension than either benzodiazepines, phenytoin, valproate, or levetiracetam. Long-acting barbiturate with sedative, hypnotic, and anticonvulsant properties (?receptor?).

propofol: Propofol is a short-acting, lipophilic intravenous general anesthetic. The drug is unrelated to any of the currently used barbiturate, opioid, benzodiazepine, arylcyclohexylamine, or imidazole intravenous anesthetic agents. Propofol causes global CNS depression, presumably through agonism of GABAA receptors and perhaps reduced glutamatergic activity through NMDA receptor blockade.

b)????

c)

GABA agonists or positive allosteric modulators increase frequency of Cl channel opening, therefore higher concentration of Cl inside cell, therefore higher action potential, therefore lower chance of firing.

wikipedia: There are three receptors of the gamma-aminobutyric acid. The two receptors GABA-α and GABA-ρ are ion channels that are permeable to chloride ions which reduces neuronal excitability. The GABA-β receptor belongs to the class of G-Protein coupled receptors that inhibit adenylyl cyclase, therefore leading to decreased cyclic adenosine monophosphate (cAMP). GABA-α and GABA-ρ receptors produce sedative and hypnotic effects and have anti-convulsion properties. GABA-β receptors also produce sedative effects. Furthermore, they lead to changes in gene transcription.

Many commonly used sedative and anxiolytic drugs that affect the GABA receptor complex aren’t agonists. These drugs are known as positive allosteric modulators (PAMs) and while they do bind to the GABA receptors, they cannot induce a response from the neuron without an actual agonist being present. Drugs that fall into this class exert their pharmacodynamic action by increasing the effects that an agonist has when potentiation is achieved.

Most general anaesthetics are PAMs of GABA-A receptor. Positive allosteric modulators work by increasing the frequency with which the chloride channel opens when an agonist binds to its own site on the GABA receptor. The resulting increase in the concentration of Cl− ions in the postsynaptic neuron immediately hyperpolarizes this neuron, making it less excitable and thus inhibiting the possibility of an action potential. However, some general anesthetics like propofol and high doses of barbiturates may not only be positive allosteric modulators of GABA-A receptors but also direct agonists of these receptors.

Na channel - phenytoin, valproic, lacosamide

GABA - propofol, phenobarbit

Ca channel - phenobarb

35
Q

Patient with frostbite injury to legs.

a. What is one reason why he may need to be admitted to the ICU?
b. List two tests the patient will require?

A

a)

tPA infusion, cardiac/hemodynamic instability

watch for compartment syndrome

b)

technetium 99 bone scan vs CT angio

???don’t know the other one…?ECG

Frostbite:

Once the patient has reached the hospital, definitive care consists of rapid rewarming, wound care, efforts to enhance tissue viability, and prevention of complications. Among patients with severe frostbite, timely diagnosis and treatment is essential to maximize limb and digit salvage. Note that the management of other, more serious conditions, such as severe hypothermia or internal hemorrhage from major trauma, takes priority over the treatment of frostbite.

Rewarming: Rewarming is most effectively accomplished by immersing the affected area in water heated to 37 to 39°C. Thawing is usually complete when the tissue is red or purple and soft to the touch. This usually takes 15 to 30 minutes. Rewarming of frostbitten tissue may be painful. Appropriate analgesia, generally opioids, should be administered.

Thrombolysis: Thrombolysis for severe injury presenting within 24 hours — Because frostbite is associated with vascular thrombosis of affected tissue, administration of intravenous or intra-arterial tissue plasminogen activator (tPA), along with intravenous or intra-arterial heparin or enoxaparin, has been used for therapy.

In patients at high risk for life-altering amputation (eg, multiple digits, proximal amputation), without contraindications to the use of tPA, who present within 24 hours of injury, we suggest treatment with intra-arterial tPA plus intra-arterial heparin. In facilities not capable of using intra-arterial tPA, we suggest using intravenous tPA, and intravenous heparin.

Patients appropriate for treatment with tPA are also appropriate for treatment with iloprost.

The degree of frostbite should be assessed using the extent of cyanosis immediately after rewarming, as described in the classification section [10]. Patients with cyanosis proximal to the interphalangeal joints should be evaluated with technetium scanning (triple phase bone scan) or CT angiography. If circulation is absent proximal to the interphalangeal joints, tPA should be given, unless contraindicated.

36
Q

A drug is lipophilic, highly protein bound, with a small volume of distribution. It is given orally crushed through an NG tube and it is excreted 100% unchanged in the urine. How will the drug concentration change in each of the following scenarios, and why?

For each scenario state if it increases/decreases and why.

  1. Positive pressure ventilation in a patient who is volume deplete
  2. Continuous renal replacement therapy
  3. Hypoalbuminemia
  4. Hypercapnia
A

1) PPV in volume deplete pt would decrease cardiac preload and therefore cardiac output and subsequent renal perfusion. This might increase the drug concentration due to decreased elimination.
2) CRRT would have a small effect on decreasing drug concentration since it is protein bound (less clearance) despite it’s small volume of distribution (which typically would be cleared sooner).
3) drug concentration would increase substantially because there is less protein (albumin) to bind to.

4)

37
Q

Patient with schizophrenia on depot haloperidol and lithium. He obsessively drinks 22L of fluid a day. He lives in a small farmhouse and is found unresponsive at home by a friend. When he presents to the emergency department his sodium is 106mEq/L. After two days in hospital he deteriorates, his Na is 126mEq/L and he is not verbalizing. He has muscle rigidity with contractions of his hands and feet and opisthotonus. What are four differential diagnoses.

A

if had to choose 4 most likely: NMS, ODS, tetanus, seizure…but might also consider meningitis/ecephalitis

NMS - Neuroleptic malignant syndrome (NMS) is a life-threatening neurologic emergency associated with the use of antipsychotic (neuroleptic) agents and characterized by a distinctive clinical syndrome of mental status change, rigidity, fever, and dysautonomia.

NMS is defined by its association with a class of medications that block dopamine transmission and a tetrad of distinctive clinical features: fever, rigidity, mental status changes, and autonomic instability.

Typical symptoms — The tetrad of NMS symptoms typically evolves over one to three days. Each feature is present in 97 to 100 percent of patients:

  • Mental status change is the initial symptom in 82 percent of patients [38]. It is not surprising, given the usual psychiatric comorbidity of the typical patient, that its significance is often underappreciated. This often takes the form of an agitated delirium with confusion rather than psychosis. Catatonic signs and mutism can be prominent. Evolution to profound encephalopathy with stupor and eventual coma is typical [17].
  • Muscular rigidity is generalized and is often extreme. The increased tone can be demonstrated by moving the extremities and is characterized by “lead-pipe rigidity” or stable resistance through all ranges of movement. Superimposed tremor may lead to a ratcheting quality or a cogwheel phenomenon. Other motor abnormalities include tremor (seen in 45 to 92 percent), and less commonly, dystonia, opisthotonus, trismus, chorea, and other dyskinesias [3,5]. Patients can also have prominent sialorrhea, dysarthria, and dysphagia.
  • Hyperthermia is a defining symptom according to many diagnostic criteria. Temperatures of more than 38°C are typical (87 percent), but even higher temperatures, greater than 40°C, are common (40 percent) [5]. Fever may be a less consistent symptom in patients with NMS associated with second-generation antipsychotic agents.
  • Autonomic instability typically takes the form of tachycardia (in 88 percent), labile or high blood pressure (in 61 to 77 percent), and tachypnea (in 73 percent) [3,21]. Dysrhythmias may occur. Diaphoresis is often profuse.

Tetanus: The most common and severe clinical form of tetanus is generalized tetanus. The presenting symptom in more than half of such patients is trismus (lockjaw), although patients with generalized tetanus sometimes present with cephalic or localized tetanus. Patients with generalized tetanus typically have symptoms of autonomic overactivity that may manifest in the early phases as irritability, restlessness, sweating, and tachycardia. In later phases of illness, profuse sweating, cardiac arrhythmias, labile hypertension or hypotension, and fever are often present.

Patients with tetanus may develop reflex spasms of their masseter muscles rather than a (normal) gag response when their posterior pharynx is touched with a tongue blade or spatula (the spatula test). In one study of 400 consecutive patients with suspected tetanus, the sensitivity and specificity of this maneuver were high (94 and 100 percent, respectively).

Patients with generalized tetanus characteristically have tonic contraction of their skeletal muscles and intermittent intense muscular spasms. Since patients with tetanus have no impairment of consciousness or awareness, both the tonic contractions and spasms are intensely painful. Tetanic spasms may be triggered by loud noises or other sensory stimuli such as physical contact or light. Tonic and periodic spastic muscular contractions are responsible for most of the classic clinical findings of tetanus such as:

  • Stiff neck
  • Opisthotonus
  • Risus sardonicus (sardonic smile)
  • A board-like rigid abdomen
  • Periods of apnea and/or upper airway obstruction due to vise-like contraction of the thoracic muscles and/or glottal or pharyngeal muscle contraction, respectively
  • Dysphagia

During generalized tetanic spasms, patients characteristically clench their fists, arch their back, and flex and abduct their arms while extending their legs, often becoming apneic during these dramatic postures.

Osmotic demyelination syndrome - The clinical manifestations of ODS are typically delayed for two to six days after overly rapid elevation of the serum sodium concentration has occurred. The symptoms, which are often irreversible or only partially reversible, include dysarthria, dysphagia, paraparesis or quadriparesis, behavioral disturbances, movement disorders, seizures, lethargy, confusion, disorientation, obtundation, and coma.

Severely affected patients may become “locked in”; they are awake but are unable to move or verbally communicate. They can usually move their eyes and blink. In patients with pontine involvement, speech abnormalities occur early and persist, and patients often become mute. Corticospinal signs (hyperreflexia and bilateral Babinski signs) and corticobulbar signs (brisk jaw jerk) are common. Swallowing dysfunction may lead to aspiration pneumonia and respiratory failure.

Other common physical findings include increased muscle tone, facial weakness, and snout, grasp, or rooting reflexes. Extrapontine involvement can result in a variety of findings, including psychiatric disturbances, catatonia, postural limb tremor, myoclonic jerks, and a parkinsonian picture with choreoathetosis or dystonia that responds to dopaminergic treatment.

Tardive Dyskinesia with tardive dystonia -

Tardive dyskinesia (TD) is a medication-induced hyperkinetic movement disorder associated with the use of dopamine receptor-blocking agents, including antipsychotic drugs and two antiemetic agents, metoclopramide and prochlorperazine. TD encompasses a wide range of abnormal, involuntary movements that often persist after discontinuation of the causative medication. TD can be irreversible and lifelong. The condition can be disfiguring and disabling, with major negative impacts on psychologic health and quality of life.

Tardive dystonia refers to TD in which dystonic manifestations predominate. Dystonia is sustained or repetitive muscle contractions that result in twisting and repetitive movements or abnormal fixed postures. Types of dystonia that occur as a manifestation of TD include retrocollis (which may be sustained or jerky), torticollis, opisthotonus, shoulder dystonia, hyperextension of the arms or legs, blepharospasm, and jaw dystonia.

Tardive dystonia occurs more frequently in patients younger than age 40; it may have a lower spontaneous remission rate than TD.

Meningitis -

Seizure -

38
Q

A 70M with COPD was admitted to hospital 1 month ago presents with diarrhea and septic shock. He has renal failure. CT abdomen shows pseudomembranes. They show you an endoscopy picture. What is the MOST important next step?

A

call surgery

pseudomembranous colitis is caused by C difficile

  • Vanco 500mg PO QID +/- Vanco retention enema if ileus present
  • Metronidazole 500mg IV Q8H
  • low threshold to consider colectomy
  • ???which of these is the MOST important step???

Patients with severe or fulminant colitis should receive antibiotic therapy, supportive care, and close monitoring. Surgery should be considered if the patient’s clinical status fails to improve within the first few days of therapy and/or the serum lactate is ≥2.2 mmol/L [72]. Toxic megacolon should be suspected if the patient develops abdominal distention with diminution of diarrhea; this may reflect paralytic ileus resulting from loss of colonic muscular tone.

Antibiotic therapy — Antibiotics for the treatment of severe CDI include oral vancomycin or oral fidaxomicin (table 2).

Antibiotics for the treatment of fulminant CDI include the combination of enteral vancomycin plus parenteral metronidazole (table 2) [1,8,12,76]. For patients with concomitant ileus (and/or another condition preventing oral vancomycin from reaching the colon), rectal vancomycin may be administered as a retention enema, either in addition to oral vancomycin (if the ileus is partial) or in place of oral vancomycin (if the ileus is complete) (table 2) [8,77-79]. However, administration of rectal vancomycin is associated with risk of colonic perforation; therefore, use of intracolonic vancomycin should be restricted to patients who are unresponsive to standard therapies, and the procedure should be performed by personnel with expertise in administering enemas [8,77-79].

The standard duration of antibiotic therapy for C. difficile diarrhea is 10 days; the antibiotic course should be tailored to clinical circumstances for patients with severe disease. Those with an underlying predisposing infection requiring prolonged duration of concomitant antibiotic administration should continue CDI treatment throughout the antibiotic course plus one additional week after its completion.

Surgery — Early surgical consultation is warranted for patients with CDI who meet one or more of the following clinical indicators that have been associated with poor prognosis:

  • Hypotension
  • Fever ≥38.5°C
  • Ileus or significant abdominal distention
  • Peritonitis or significant abdominal tenderness
  • Altered mental status
  • White blood cell count ≥20,000 cells/mL
  • Serum lactate levels >2.2 mmol/L
  • Admission to intensive care unit
  • End organ failure (eg, requiring mechanical ventilation, renal failure)
  • Failure to improve after three to five days of maximal medical therapy

IDSA guidelines on CDiff

XXVIII. What are important ancillary treatment strategies for CDI?

Discontinue therapy with the inciting antibiotic agent(s) as soon as possible, as this may influence the risk of CDI recurrence (strong recommendation, moderate quality of evidence).

Antibiotic therapy for CDI should be started empirically for situations where a substantial delay in laboratory confirmation is expected, or for fulminant CDI (described in section XXX) (weak recommendation, low quality of evidence).

XXIX. What are the best treatments of an initial CDI episode to ensure resolution of symptoms and sustained resolution 1 month after treatment?

Either vancomycin or fidaxomicin is recommended over metronidazole for an initial episode of CDI. The dosage is vancomycin 125 mg orally 4 times per day or fidaxomicin 200 mg twice daily for 10 days (strong recommendation, high quality of evidence) (Table 1).

In settings where access to vancomycin or fidaxomicin is limited, we suggest using metronidazole for an initial episode of nonsevere CDI only (weak recommendation, high quality of evidence). The suggested dosage is metronidazole 500 mg orally 3 times per day for 10 days. Avoid repeated or prolonged courses due to risk of cumulative and potentially irreversible neurotoxicity (strong recommendation, moderate quality of evidence). (See Treatment section for definition of CDI severity.)

XXX. What are the best treatments of fulminant CDI?

For fulminant CDI*, vancomycin administered orally is the regimen of choice (strong recommendation, moderate quality of evidence). If ileus is present, vancomycin can also be administered per rectum (weak recommendation, low quality of evidence). The vancomycin dosage is 500 mg orally 4 times per day and 500 mg in approximately 100 mL normal saline per rectum every 6 hours as a retention enema. Intravenously administered metronidazole should be administered together with oral or rectal vancomycin, particularly if ileus is present (strong recommendation, moderate quality of evidence). The metronidazole dosage is 500 mg intravenously every 8 hours.*

*Fulminant CDI, previously referred to as severe, complicated CDI, may be characterized by hypotension or shock, ileus, or megacolon.

If surgical management is necessary for severely ill patients, perform subtotal colectomy with preservation of the rectum (strong recommendation, moderate quality of evidence). Diverting loop ileostomy with colonic lavage followed by antegrade vancomycin flushes is an alternative approach that may lead to improved outcomes (weak recommendation, low quality of evidence).

39
Q

ABG: 7.46/24/80/16. Anion gap 22.

a. What is the ABG abnormality
b. What two conditions might give you this abnormality?

A

a) respiratory alkalosis with chronic metabolic compensation (but I think they are looking for additional metabolic acidosis)

b)

  • ASA toxicity
  • theophylline
  • https://www.mcgill.ca/criticalcare/teaching/files/toxicology/theophylline
  • ?sepsis causing metabolic acidosis and pain/delirium causing hyperventilation???
40
Q

Bronchoscopy pictures – identify the bronchus (three images)

A

review video on ATS here

Toronto PIE website here

good place to test yourself here

Trachea

  • right mainstem bronchus
    • upper lobe bronchus
      • apical segmental bronchus
      • posterior segmental bronchus
      • anterior segmental bronchus
    • bronchus intermedius
      • right middle lobe bronchus (really an extension of the bronchus intermedius)
        • lateral segmental bronchus
        • medial segmental bronchus
      • right lower lobe bronchus
        • superior
        • medial
        • anterior
        • posterior
        • lateral
  • left mainstem bronchus
    • upper lobe bronchus
      • apical
      • posterior
      • anterior
      • superior lingular
      • inferior lingular
    • lower lobe bronchus
      • lateral
      • posterior
      • anterior
      • medial
      • superior
41
Q

70M undergoes aortic valve replacement. His TEE postoperatively showed normal LV function. He requires pacing, but when paced his cardiac output drops by 20%. A Rhythm strip is given that shows some spikes without capture, and some spikes right after a QRS. What three changes can you make to the pacemaker to improve cardiac output?

A
  1. increase pacing amperage (so it will capture mechanical beats)
  2. decrease the sensitivity threshold (so it can sense intrinsically paced beats)
  3. increase pacing rate (higher HR = higher CO, most of the time)
42
Q

Vasculopath lady with a history of intestinal angina postprandial pain and nausea and HTN etc. Presents with worsening abdominal pain:
a. What are the three vessels that supply the bowel and which parts do they supply

b. You already know she has symptoms of ischemia from occlusion of which vessel?
c. What is wrong with these CT and angiogram images?
d. What is the next most important step?

from another version:

Old lady with abdominal angina, hx AobiFem bx 7 years ago. Imaging of coronal CT through aorta.

a. What 3 vessels perfuse the gut and describe the distribution
b. Based on her PMHx, what’s vessel is already compromised
c. Stem – 2 days later, inc abdo pain, lactate elevated, hypotension, angiogram done.
i. What is most important finding – SMA thrombus?
ii. What immediate treatment

A

a) celiac trunk -> stomach, spleen, liver, GB, abdominal esophagus, pancreas, duodenum

superior mesenteric artery -> duodenum to proximal 2/3rds of the transverse colon

inferior mesenteric artery -> distal 1/3rd of the transverse colon, splenic flexure, descending colon, sigmoid colon, rectum

b) ?which vessel is already compromised? I guess Aorta and/or iliac/femoral arteries

c)

d) treatment of SMA thrombus

Initial medical management for all patients with acute mesenteric ischemia includes the following, which are discussed in detail separately:

  • Nothing by mouth, nasogastric decompression.
  • Fluid therapy to maintain adequate intravascular volume and visceral perfusion and monitored as normal urine output.
  • Avoidance of vasopressors, which can exacerbate ischemia.
  • Antithrombotic therapy consists of anticoagulation (unfractionated heparin, weight-based protocol) to limit thrombus propagation and help alleviate associated arteriolar vasoconstriction with or without antiplatelet therapy [32].
  • Empiric broad-spectrum antibiotic therapy.
  • Proton pump inhibitors [33].
  • Supplemental oxygen [33].

Approach to treatment — Clinical evaluation and vascular imaging determine whether the patient is a candidate for vascular intervention and whether the occlusion is embolic or thrombotic (algorithm 2 and algorithm 3), which has a bearing on the type of intervention offered. The goal of vascular intervention is to restore intestinal blood flow as rapidly as possible. The specific treatment chosen depends upon the clinical status of the patient and the etiology and location of the occlusion. Optimal treatment may include open, endovascular, or a combined approach. The ability to offer an endovascular approach depends upon local resources and the availability of vascular specialists. A hybrid interventional suite/operating room may be the ideal setting to manage acute mesenteric arterial occlusion, but these are generally available only at large vascular centers.

Some patients (eg, acute-on-chronic occlusion) who are hemodynamically stable and do not have clinical signs of advanced bowel ischemia can be observed while on heparin anticoagulation, if there is evidence of good collateral blood flow on vascular imaging studies. Antiplatelet therapy may be justified in this setting if the risk of progressive ischemia appears to be greater than the risk of bleeding [32,34]. The patient should have serial clinical assessment (laboratory, physical examination) with a low threshold to repeat abdominal imaging studies or, if abdominal symptoms progress, surgical or endovascular intervention. (See ‘Surgical versus endovascular intervention’ below.)

A palliative approach may be the best option for poor-risk surgical candidates with extensive transmural infarction (eg, small bowel up to the midtransverse colon). Extensive bowel resection would be inappropriate for these patients and may also be inappropriate for a subset of patients who might otherwise be expected to tolerate the procedure but for whom lifelong parenteral nutrition would be unacceptable

Patients who are good-risk surgical candidates with indications for immediate laparotomy such as peritonitis or radiologic features of advanced bowel ischemia (free air, extensive pneumatosis) should be taken directly to the operating room for exploration. Resection of bowel should ideally be delayed until after mesenteric arterial revascularization can be performed to salvage as much bowel as possible; however, in practice, this sequence does not commonly occur. In situations where an individual with appropriate vascular expertise is not immediately available, resection of grossly necrotic or perforated bowel (leaving any questionable bowel) while awaiting intraoperative consultation is appropriate, or, alternatively, following resection, abdominal closure and transfer is also a reasonable option when required.

  • The traditional treatment for mesenteric embolism is open surgical embolectomy, which, in addition to expeditiously clearing the thrombus, allows direct assessment of bowel viability.
  • Open surgical treatment of mesenteric artery thrombosis is treated principally with mesenteric bypass. Thrombectomy alone is unlikely to offer a durable solution due to the presence of thrombogenic atherosclerotic plaques. Intraoperative retrograde superior mesenteric artery angioplasty and stenting is another option, particularly in the presence of gross contamination where bypass is more problematic.

Patients who are hemodynamically stable and who do NOT have clinical or radiologic signs of advanced intestinal ischemia may be candidates for a primary endovascular approach.

43
Q

28F involved in an MVC with epidural hematoma and DAI. According to the Munroe-Kelly doctrine, what are the four components of ICP and what steps can you make to improve her ICP:

A
  1. brain tissue (80%)
  2. csf (10%)
  3. arterial blood volume (10% together with venous blood)
  4. venous blood volume
  5. +/-mass effect
  • raise head of bed (decreases venous blood volume)
  • mannitol/hypertonic saline (decreases brain tissue edema)
  • EVD insertion (decreases CSF)
  • drain epidural hematoma
  • sedation/paralysis/cooling but not sure how it applies to Munroe-Kelly
  • +/- decompressive craniectomy
44
Q

G1P1 at 35 weeks is admitted to the ICU following minor trauma or something.

a. What percentage of cardiac output goes to the uterus at this stage in pregnancy?
b. How will you monitor blood flow to the uterus?
c. What two maternal or fetal things would require an urgent gyne consult?

A

a)

ATLS 20% at third trimester

uptodate: Both invasive and noninvasive measures have estimated uteroplacental blood flow between 450 and 750 mL/minute at term [69]. For comparison, approximately 5000 mL/minute flows through the entire circulation of a nonpregnant woman. —> this suggests ~10%

book chapter by stephen lapinsky states CO increases 30-50% by 28wks

old article says 3.5% at beginning of pregnancy increases to 12% at term

sketchy OBGYN blog post says it’s 17% at term

normal changes during pregnancy:

Plasma volume increases progressively throughout normal pregnancy.2 Most of this 50% increase occurs by 34 weeks’ gestation and is proportional to the birthweight of the baby. Because the expansion in plasma volume is greater than the increase in red blood cell mass, there is a fall in haemoglobin concentration, haematocrit and red blood cell count. Despite this haemodilution, there is usually no change in mean corpuscular volume (MCV) or mean corpuscular haemoglobin concentration (MCHC).

The platelet count tends to fall progressively during normal pregnancy, although it usually remains within normal limits. In a proportion of women (5–10%), the count will reach levels of 100–150 × 109 cells/l by term and this occurs in the absence of any pathological process. In practice, therefore, a woman is not considered to be thrombocytopenic in pregnancy until the platelet count is less than 100 × 109 cells/l.

Changes in the cardiovascular system in pregnancy are profound and begin early in pregnancy, such that by eight weeks’ gestation, the cardiac output has already increased by 20%. The primary event is probably peripheral vasodilatation. This is mediated by endothelium-dependent factors, including nitric oxide synthesis, upregulated by oestradiol and possibly vasodilatory prostaglandins (PGI2). Peripheral vasodilation leads to a 25–30% fall in systemic vascular resistance, and to compensate for this, cardiac output increases by around 40% during pregnancy. This is achieved predominantly via an increase in stroke volume, but also to a lesser extent, an increase in heart rate. The maximum cardiac output is found at about 20–28 weeks’ gestation. There is a minimal fall at term.

Placental circulation is unique in having no capillary microcirculation but a high flow, low resistance system of spiral arteries. The uteroplacental vascular bed is usually maximally dilated, but has a strong alpha-adrenergic receptor system with a marked decrease in uteroplacental blood flow in response to endogenous or exogenous stimulation [1]. Normal maternal hemodynamic responses to hypotension are therefore not protective of the fetus, but rather protect maternal vital organs, shifting cardiac output from the uterine and placental circulation to the maternal brain and heart. Fetal oxygenation decreases as placental perfusion (and therefore oxygen delivery) decreases.

Clinical data on the use of these agents is limited, although animal studies provide some information. Norepinephrine, epinephrine and dopamine have all been shown to reduce uterine blood flow [3]. Ephedrine and phenylephrine have been used for maternal hypotension secondary to neuraxial anesthesia, in small bolus doses or by infusion. Phenylephrine is currently the preferred option [4,5]. Left lateral tilt of the hypotensive pregnant women should always be considered as an early intervention, to reverse a supine hypotensive effect [6]. Our practice with regard to vasopressor use in pregnancy is to follow our usual protocols (that is, most commonly norepinephrine with or without dobutamine) after adequate fluid resuscitation and left lateral positioning. In consultation with the obstetric team electronic fetal heart rate monitoring (see below) may be utilized to identify whether these infusions are producing beneficial or adverse effects on the fetus.

c)

from the guidelines:

  1. In cases of major trauma, the assessment, stabilization, and care of the pregnant women is the first priority; then, if the fetus is viable (≥ 23 weeks), fetal heart rate auscultation and fetal monitoring can be initiated and an obstetrical consultation obtained as soon as feasible. (II-3B)
  2. In pregnant women with a viable fetus (≥ 23 weeks) and suspected uterine contractions, placental abruption, or traumatic uterine rupture, urgent obstetrical consultation is recommended. (II-3B)
  3. In cases of vaginal bleeding at or after 23 weeks, speculum or digital vaginal examination should be deferred until placenta previa is excluded by a prior or current ultrasound scan. (III-C)
45
Q

Name the four types of tissue hypoxia and briefly describe each.

A
  1. hypoxemia - low oxygen content in arterial blood
  2. anemia - amount of functional Hgb is too small and hence capacity to carry oxygen is too low
  3. stagnant type - blood flow is reduced or unevenly distributed
  4. histotoxic type - cells are unable to use the oxygen, characteristically produced by cyanide but can be done by any agent that decreases cellular respiration including narcotics, alcohol, formaldehyde, acetone, and certain anesthetic agents
46
Q

With regards to SAH and vasospasm:

a. List two treatments to prevent vasospasm
b. List two treatments for established vasospasm
c. What is the ONE most important predictor of vasospasm
d. List two treatments for refractory vasospasm

A

a)

would you say fluid status and statins or nimodipine…I’m tempted to say statins as my second half

  • maintenance of euvolemic/positive fluid balance
  • statins:

While the mechanism by which statins might reduce vasospasm is uncertain, it is believed that these agents have the potential to improve cerebral vasomotor reactivity by upregulating endothelial nitric oxide synthase, increase cerebral blood flow, and attenuate vasculopathy [82-85].

It remains uncertain whether statin therapy reduces the incidence of vasospasm, delayed cerebral ischemia, poor neurologic outcome, or mortality. While findings from at least three small randomized placebo-controlled trials suggested that statin tre

-nimodipine 60mg PO q4H for 21d after SAH (ideally start within 4 days of SAH) - DO NOT give IV because it’s been associated with complications.

***The calcium channel blocker nimodipine was initially studied in patients with SAH as a means to prevent vasospasm. However, despite the vasodilatory effects of nimodipine on cerebral vessels, there is no convincing evidence that nimodipine affects the incidence of either angiographic or symptomatic vasospasm [63-68]. Nevertheless, nimodipine has been demonstrated to improve outcomes in SAH and is the standard of care in these patients

b)

  • BP augmentation
  • IVF

Aggressive therapy of vasospasm can only be pursued after the aneurysm has been treated with surgery or intraluminal therapy. Following aneurysmal occlusion, treatment options include:

  • Hemodynamic augmentation has not been evaluated in randomized clinical trials; however, in case series of patients with symptomatic vasospasm, this approach has been associated with improved cerebral blood flow and reversal of ischemia in more than half of patients [95-97].
  • Traditionally this approach, so-called “triple-H” therapy, included modest hemodilution, induced hypertension and hypervolemia instituted in an effort to raise the mean arterial pressure and thereby increase cerebral perfusion. More recently, the focus has shifted toward maintenance of euvolemia using crystalloid or colloid solution and induced hypertension with vasopressor agents
  • The addition of inotropic support with agents such as dobutamine or milrinone have been reported to be helpful in patients who do not appear to respond to pressors alone

c)

-amount of blood/grade of SAH

One of the best predictors of cerebral vasospasm to date is the characteristic of aneurysmal SAH on admission CT shortly after aneurysmal rupture of the aneurysm [33, 34]. The most commonly used and well-known grading scale for classification of admission CT is the Fisher grading scale [35, 36]. In the well-known analysis between the amount of subarachnoid blood and development of cerebral vasospasm, Fisher et al. found that there was a high correlation between symptomatic vasospasm and Grade III SAH.

Some of the strongest predictors of vasospasm relate to the severity of the aSAH as graded according to several well-accepted classification systems. Both the Fisher Scale and the Hunt-Hess grading system have been shown to correlate with the incidence of vasospasm.

On the Fisher Scale, the thickness of the aSAH clot and the presence of intraventricular hemorrhage (IVH) are strong predictors of vasospasm. [8, 9] These are the primary determinants of the Modified Fisher Scale, with a higher Fisher score being directly correlated with the risk of vasospasm.

Cigarette smoking also appears to increase the risk of angiographic vasospasm and delayed cerebral ischemia in patients with ruptured aSAH.

d)

  • milrinone (intra-arterial)
  • angioplasty +/- stenting
  • Balloon angioplasty has become the mainstay of treatment at many centers for symptomatic focal vasospasm of the larger cerebral arteries which is refractory to hemodynamic augmentation, again despite an absence of clinical trial data
  • Intra-arterial administration of vasodilators are generally used for diffuse vasospasm involving smaller arterial branches.
47
Q

a) List four components of the central line insertion bundle:
b) List four components of the central line care bundle
c) List the metric for measuring central line infections

A

a)

  • perform hand hygiene before insertion
  • adhere to aseptic technique
  • use maximal sterile barrier precautions (mask, cap, gown, gloves, full body drape)
  • choose the best insertion site to minimize infections and noninfectious complications based on individual patient characteristics (avoid femorals in obese pts)
  • >0.5% chlorhexidine with alcohol
  • sterile gauze dressing or sterile transparent semipermeable dressing over site
  • For patients 18 years of age or older, use a chlorhexidine impregnated dressing with an FDA cleared label that specifies a clinical indication for reducing CLABSI for short term non-tunneled catheters unless the facility is demonstrating success at preventing CLABSI with baseline prevention practices

b)

  • Comply with hand hygiene requirements
  • Bathe ICU patients over 2 months of age with a chlorhexidine preparation on a daily basis
  • Scrub the access port or hub with friction immediately prior to each use with an appropriate antiseptic (chlorhexidine, povidone iodine, an iodophor, or 70% alcohol)
  • Use only sterile devices to access catheters
  • Immediately replace dressings that are wet, soiled, or dislodged
  • Perform routine dressing changes using aseptic technique with clean or sterile gloves.
    • Change gauze dressings at least every two days or semipermeable dressings at least every seven days.
    • For patients 18 years of age or older, use a chlorhexidine impregnated dressing with an FDA cleared label that specifies a clinical indication for reducing CLABSI for short-term non-tunneled catheters unless the facility is demonstrating success at preventing CLABSI with baseline prevention practices.
  • Change administrations sets for continuous infusions no more frequently than every 4 days, but at least every 7 days.
    • If blood or blood products or fat emulsions are administered change tubing every 24 hours.
    • If propofol is administered, change tubing every 6-12 hours or when the vial is changed

c)

Catheter-related blood stream infection per 1000 catheter days

48
Q

You are given a left ventricular diastolic pressure volume graph with pressure on the y axis and volume on the x axis. They have drawn the normal pressure volume trace. They then ask you to draw a trace for:

a. Dilated cardiomyopathy
b. Tamponade

A

DCM

DCM increases end-diastolic volume (EDV) and end-systolic volume (ESV). However, the weakening of the heart causes a decrease in end-systolic pressure (ESP). Thus, the slope of the end-systolic pressure volume relationship (ESPVR) decreases. The slope of the end-diastolic pressure volume relationship (EDPVR) decreases which signifies an increase in compliance (decrease in vascular stiffness). Stroke work and cardiac output are also depressed.

TamponadeDenault article

Mechanism of pericardial tamponade. a The normal venous return (VR) curve and cardiac output (CO) using the Starling relationship. Both VR and CO are equal at a specific right atrial pressure (Pra). The venous return is equal to the difference between the mean systemic pressure (Pms) and the Pra divided by the resistance to VR. The resistance to VR is a function of the slope of the VR. At a certain point on this curve, VR cannot increase because Pra becomes sub-atmospheric. This leads to collapse of the great vessels entering the intra-thoracic cavity. The corresponding normal pressure–volume relationship of the right and left ventricles are shown below. b Top In tamponade, VR and CO are reduced and Pra is increased. This is secondary to the rise in pericardial pressure. In addition, VR will now be limited by the pericardial pressure, not by the sub-atmospheric pressure. Therefore, VR is now equal to the difference between Pms and pericardial pressure divided by the resistance to VR. The VR slope is reduced by an increase in the resistance to VR. A normal compensatory increase in mean systemic pressure (Pms) will also be observed secondary to the activation of the autonomic nervous system. Bottom Bi-ventricular pressure–volume relationships in pericardial tamponade. The increase in pericardial pressure will be transmitted to both ventricles. Therefore, an upward shift of the horizontal part of the pressure–volume relationship will be observed. This is typically associated with equalization of end-diastolic pressures. As pericardial pressure increases and tamponade develops, bi-ventricular volumes will be further reduced. Consequently, left ventricular pressure and systemic pressure will be reduced. (see image in question)

49
Q

Describe how an SpO2 probe works.
a. What are three reasons why the reading would not be reliable

A

Pulse oximetry uses spectrophotometry to determine the proportion of hemoglobin that is saturated with oxygen (ie, oxygenated hemoglobin; oxyhemoglobin) in peripheral arterial blood. Light, at two separate wavelengths, illuminates oxygenated and deoxygenated hemoglobin in blood. The ratio of light absorbance between oxyhemoglobin and the sum of oxyhemoglobin plus deoxyhemoglobin is calculated and compared with previously calibrated direct measurements of arterial oxygen saturation (SaO2) to establish an estimated measure of peripheral arterial oxygen saturation (SpO2) [4].

Probes — Pulse oximeter probes consist of two light-emitting diodes and a photodetector.

  • Emitters – Deoxyhemoglobin absorbs light maximally in the red band of the spectrum (600 to 750 nm), and oxyhemoglobin absorbs maximally in the infrared band (850 to 1000 nm) [1]. Thus, the emitters emit light at 660 nm and 940 nm for optimal detection of these two substances.
  • Detector – The detector (also known as sensor) detects the absorbance of light from exposed tissue. The values are processed and a saturation determined.

The saturation value is calculated using microprocessors that utilize absorbance readings from light-exposed oxyhemoglobin and deoxyhemoglobin. The emitters are switched on and off several hundred times per second [19]. Absorption during pulsatile flow relates to the characteristics of arterial blood plus background tissue and venous blood, whereas absorption during nonpulsatile flow is due only to the background tissue and venous blood.

The microprocessors of pulse oximeters are calibrated using reference tables of actual SaO2 measurements performed using co-oximetry and compiled using data from exposing healthy volunteers to decreasing fraction of inspired oxygen (FiO2) to yield SaO2 ranging from 100 to 75 percent. Because it would be unethical to intentionally generate lower saturations in volunteers, values for an SaO2 less than 75 percent are obtained by extrapolation from these volunteer data. Pulse oximeter manufacturers claim that reported values between 70 and 100 percent are accurate to within ± 2 percent of the true value, while those between 50 and 70 percent are within ± 3 percent.

a)

  • motion/noise artifact (siezure, shivering, pt transport)
  • hypoperfusion (shock, vasoconstriction, PVD, In adults, the accuracy of standard pulse oximeters decreases dramatically when systolic blood pressure falls below 80 mmHg, generally resulting in underestimation of the actual arterial oxygen saturation)
  • hypothermia
  • occasionally skin pigment results in poor amplitude readings

falsely normal/high readings

  • carboxyhemoglobin (Carboxyhemoglobin absorbs at approximately the same amount of 660 nm light as oxyhemoglobin. Thus, the pulse oximetry reading represents an inexact summation of oxyhemoglobin and carboxyhemoglobin. Due to the interference of high levels of carboxyhemoglobin in carbon monoxide (CO) poisoning, or in chronic, heavy smokers, a falsely reassuring normal pulse oximetry reading may mask life-threatening arterial desaturation. Arterial oxygen tension (PaO2) measurements tend to be normal because PaO2 reflects O2 dissolved in blood, and this process is not affected by CO. In contrast, hemoglobin-bound O2(which normally comprises 98 percent of arterial O2 content) is profoundly reduced in the presence of carboxyhemoglobin.) In such cases, whenever carboxyhemoglobinemia is suspected, co-oximetry (not pulse oximetry) is recommended for the measurement of carboxyhemoglobin levels.
  • Glycohemoglobin A1c — Glycohemoglobin A1c levels greater than 7 percent in type 2 diabetics with poor glucose control have been shown to result in overestimation of arterial oxygen saturation (SaO2) by pulse oximetry.

Falsely low readings

  • Methemoglobin — Methemoglobin absorbs at both 660 and 940 nm [7]. Methemoglobinemia should be suspected in those with cyanosis and normal PaO2. Up to a methemoglobin level of 20 percent, SaO2 drops by about one-half of the methemoglobin percentage. At higher methemoglobin levels, SaO2 trends toward 85 percent regardless of the true percentage of oxyhemoglobin, thus leading to over- or under-estimation of the true SaO2 [4,9,71,72]. When methemoglobinemia is suspected, co-oximetry should be used to accurately determine the methemoglobin level.
  • Sulfhemoglobin — Sulfhemoglobin absorbs at 660nm, similar to oxyhemoglobin, and its absorbance at 940 nm is unknown. Sulfhemoglobinemia is most commonly caused by the ingestion of oxidizing drugs (eg, dapsone, sulfonamides, metoclopramide, nitrates) and patients present in a similar fashion to methemoglobinemia (cyanosis and normal PaO2).
  • Sickle hemoglobin — Sickle hemoglobin generally produces pulse oximeter readings similar to normal hemoglobin, but rare cases of falsely elevated and falsely low readings have been reported especially during vaso-occlusive crises (perhaps due to hypoperfusion).
  • Inherited forms of abnormal hemoglobin — Inherited forms of abnormal hemoglobin (Hb) are rare but have been reported to in falsely low SpO2 readings.
  • Severe anemia — In vitro and animal studies suggest that pulse oximetry readings may be affected by profoundly decreased hemoglobin concentration.
  • Venous congestion — Venous congestion due to tricuspid valve incompetence or cardiomyopathy may yield falsely low SaO2 readings, due to the generation of venous pulsations. This results from the instrument detecting less oxygenated, pulsatile venous blood as part of the arterial sample, thereby underestimating the actual SaO2.
  • Ambient light — Intense daylight, fluorescent, incandescent, xenon, and infrared light sources have been reported to cause spurious pulse oximetry readings.
  • Nail polish — The use of nail polish can potentially affect pulse oximeter readings if the polish absorbs light at 660 nm and/or 940 nm.
  • Vital dyes — Vital dyes, such as methylene blue (used to treat methemoglobinemia, or during endoscopic polypectomy), indocyanine green (used for measuring cardiac output, for ophthalmic angiography, or for measuring liver blood flow), fluorescein (ophthalmic angiography) and isosulfan blue (used intraoperatively to mark breast and melanoma tumors), can cause erroneously low pulse oximetry readings due to absorption of light at 660 nm or 940 nm [20,100-105]. Methylene blue has the greatest impact, as it absorbs significantly at 670 nm. However, these effects tend to be transient, and resolve rapidly as the dyes are diluted and metabolized
50
Q

A Patient is post Bentall procedure and you are worried about tamponade.

a. Define pulsus paradoxus
b. Describe two physiologic mechanisms why this happens
c. List two conditions besides tamponade where you may have an elevated pulsus paradoxus
d. List three reasons why you could have tamponade with no pulsus paradoxus

A

a) difference in systolic blood pressure >12mm Hg between inspiration and expiration

b)

  1. change in intrathoracic pressure affects venous return to heart, less venous return means less cardiac output
  2. increased pericardial pressure reduces RV compliance and thus with increased filling which happens during inspiration the RV pressure exceeds LV pressure causing bowing of the interventricular septum towards the LV and thus decreased LV filling and thus decreased LV stroke volume/cardiac output

c)

  1. obstructive lung disease (COPD or asthma)
  2. excessive respiratory efforts causing large changes in intra-thoracic pressure
  3. constrictive pericarditis
  4. RV infarct
  5. restrictive cardiomyopathy
  6. OSA
  7. PE
  8. tension pneumothorax
  9. bilateral pleural effusions
  10. marked obesity
  11. hypovolemic shock
  12. pectus excavatum

d)

  • positive pressure ventilation (although not listed on uptodate)
  • co-existing disease that elevates LV diastolic pressure (systemic HTN, AS) or RV diastolic pressure (pulm HTN with cor pulmonale)
  • intracardiac shunt with significant valvular regurgitation (AR)
  • aortic dissection with resulting periardial effusion and cardiac tamponade
  • “low pressure tamponade” as in the presence of dehydration and hypovolemia where a pericardial effusion would otherwise cause cardiac compression can affect cardiac function
51
Q

Another pregnant patient presents with stroke:
a. Aside from procoagulable state, list four reasons why a pregnant patient may
have a stroke
b. She is now seizing, list three drugs you can use in pregnancy to control seizures.

A

Taken together, pregnancy and the postpartum period are associated with a marked increase in the relative risk and a small increase in the absolute risk of ischemic stroke and intracerebral hemorrhage, with the highest risk during the puerperium.

  1. pregnancy-induced hypertension
  2. peripartum infection
  3. preeclampsia/eclampsia
  4. postpartum cerebral angiopathy
  5. gestational trophoblastic disease
  6. amniotic fluid embolism

The syndromes of severe preeclampsia, eclampsia, and HELLP (hemolysis, elevated liver enzymes, and low platelets) are among the most common causes of both ischemic and hemorrhagic stroke in pregnancy [5,6,22-24]. However, the most frequent cerebrovascular disturbance associated with preeclampsia and eclampsia is posterior reversible encephalopathy syndrome (PRES), which is also known as reversible posterior leukoencephalopathy syndrome. The presumed mechanism is impairment of cerebrovascular autoregulation.

Thrombotic thrombocytopenic purpura and hemolytic uremic syndrome — Thrombotic thrombocytopenic purpura (TTP) and hemolytic uremic syndrome (HUS) are acute syndromes with abnormalities in multiple organ systems that demonstrate microangiopathic hemolytic anemia and thrombocytopenia (table 2). Although some studies distinguish between TTP and HUS, the presenting features are similar in most patients: microangiopathic hemolytic anemia and thrombocytopenia without another apparent cause and, in many patients, neurologic and/or renal abnormalities.

Postpartum angiopathy — Postpartum angiopathy appears to be one member of a group of reversible cerebral vasoconstriction syndromes (RCVS) with similar clinical and radiologic features that are characterized by thunderclap headache and diffuse, segmental, reversible cerebral vasospasm.

Cerebral venous thrombosis — Cerebral venous thrombosis (CVT), also known as cerebral venous sinus thrombosis, is rare, but occurs more commonly in association with pregnancy [31,32]. It presents most often in the third trimester of pregnancy and the puerperium.

b)

The choice of antiepileptic drug (AED) treatment is complicated by concerns of fetal safety. The evidence linking valproate to fetal malformations and neurodevelopmental disorders is sufficiently convincing to recommend avoiding its initiation in pregnancy. Although lamotrigine is a favorable choice during preconceptional planning, it is not a good choice for initiation during pregnancy; lamotrigine cannot be started quickly due to the higher risk of rash with accelerated titration , and it is difficult to get to a therapeutic concentration due to the enhanced clearance during pregnancy. Levetiracetam is a medication with a favorable reproductive safety profile, which can be started at a therapeutic dose immediately, and which has a broad spectrum of action across multiple seizure types. If seizures are focal and begin after the first trimester, carbamazepine is another option given the data supporting normal neurodevelopmental profiles after in utero exposure

b)

?Lorazepam and its metabolite cross the human placenta. Teratogenic effects in humans have been observed with some benzodiazepines (including lorazepam); however, additional studies are needed.

?Propofol crosses the placenta and may be associated with neonatal CNS and respiratory depression.

?Phenytoin crosses the placenta. An increased risk of congenital malformations and adverse outcomes may occur following in utero phenytoin exposure.

…is this question mostly asking us to avoid valproic acid and lamotrigine

52
Q

COPD patient with dynamic hyperinflation and increased work of breathing:

a. What is the problem you see on the flow time loop?
b. What are three ventilator strategies you can use to manage this?

A

a) ?dynamic hyperinflation if the expiration loop does not return to no flow before next inspiration

b)

  1. decrease RR
  2. decrease tidal volume
  3. decrease I:E ratio
53
Q

Young man takes 30g Tylenol in a suicide attempt. His INR is 6.9 transaminases >10000 Bilirubin is up and platelets are down:

a. What are two criteria for acute hepatic failure
b. What are four consequences of acute hepatic failure
c. What are the two most common causes of death for patients with hepatic failure
d. What is the one most important prognostic finding that would indicate the patient needs a liver transplant?

A

a) Acute liver failure (<26wks) is diagnosed by demonstrating all of the following:

  • Elevated aminotransferases (often with abnormal bilirubin and alkaline phosphatase levels)
  • Hepatic encephalopathy
  • Prolonged prothrombin time (INR ≥1.5)

b)

  1. metabolic abnormalities: hypokalemia, hyponatremia, hypoglycemia, mixed resp/met alkalosis (early) then met acidosis from lactate and resp alkalosis
  2. hepatic encephalopathy
  3. cerebral edema, occurs in up to 30% of pts with grade III encephalopathy and 75% of those with grade IV encephalopathy
    • Intracranial pressure monitoring — Because of the devastating consequences of ICP elevation and the difficulty in accurately assessing for its presence based upon clinical examination alone, some suggest invasive means of monitoring ICP to guide management. However, severe complications from ICP monitoring may develop, such as infection and bleeding, and observational studies of patients undergoing ICP monitoring did not find differences in overall survival between those who received ICP monitoring and those who did not.
  4. seizures: Seizures in patients with acute liver failure should be treated promptly because seizure activity increases ICP and may cause cerebral hypoxia. Consensus recommendations suggest treatment with phenytoin, since patients with acute liver failure have a severely impaired ability to clear sedatives [2]. Patients who are refractory to phenytoin should receive short acting benzodiazepines.
  5. acute renal failure: Acute renal failure complicates acute liver failure in approximately 30 to 50 percent of patients [16,31,67]. The frequency of acute renal failure is higher (up to 75 percent) for etiologies of acute liver failure that are known to independently damage the kidneys, such as acetaminophen intoxication [11,49]. Once renal failure develops, it is usually progressive and is associated with a poor prognosis without liver transplantation.
  6. pulmonary complications: Pulmonary edema and pulmonary infections are encountered in approximately 30 percent of patients with acute liver failure [16]. Mechanical ventilation may be required to ensure adequate oxygenation. However, extreme caution must be used with positive end-expiratory pressure in patients with acute liver failure since it can worsen cerebral edema.

c)

  • sepsis and subsequent multiorgan failure
  • cerebral edema –>intracranial hypertension –> brainstem herniation

citation

d)

?high grade hepatic encephalopathy???

As a general rule, the most important factors for predicting the outcome in acute liver failure are the degree of encephalopathy (table 1), the patient’s age, and the cause of the acute liver failure. These factors in part reflect the importance of the severity of the hepatic injury and the likelihood of reversal of the underlying process either spontaneously or with specific therapy (eg, N-acetylcysteine in acetaminophen toxicity).

54
Q

What are four contraindications to liver transplant?

A
  • Cardiopulmonary disease that cannot be corrected and is a prohibitive risk for surgery
  • AIDS
  • Malignancy outside of the liver not meeting oncologic criteria for cure
  • Hepatocellular carcinoma with metastatic spread
  • Intrahepatic cholangiocarcinoma
  • Hemangiosarcoma
  • Anatomic abnormalities that preclude liver transplantation
  • Uncontrolled sepsis
  • Acute liver failure with a sustained intracranial pressure >50 mmHg or a cerebral perfusion pressure <40 mmHg
  • Persistent nonadherence with medical care
  • Lack of adequate social support
55
Q

Man with depression since his wife died four months ago. He is found unresponsive at home with bottles of verapamil, hydrochlorothiazide, and atorvastatin. His HR is 40, BP 80/40. The emergency physicians have tried volume resuscitation and atropine, and he is being transcutaneously paced without capture or improvement in BP, list four more treatment options you could try.

A
  • IC calcium
  • glucagon
  • high dose insulin
  • lipid emulsion
  • ECMO
  • norepinephrine

uptodate:

for severely symptomatic patients:

  • Stabilization of the airway as necessary (avoid induction agents that exacerbate hypotension)
  • Additional IV boluses of isotonic crystalloid
  • IV calcium salts
  • IV glucagon
  • IV high-dose insulin and glucose
  • IV vasopressor (eg, norepinephrine)
  • IV lipid emulsion therapy

Atropine should be administered to any patient with symptomatic bradycardia; however, it is often ineffective after significant CCB exposure [15]. Atropine is administered in a dose of 0.5 to 1.0 mg IV every two to three minutes to a maximum of 3 mg.

Calcium salts are often used to overcome the cardiovascular effects of CCBs. However, treatment with calcium salts is often ineffective. This is because CCB poisoning interferes with both the serum concentration and the intracellular handling of calcium.

Glucagon increases intracellular levels of cyclic adenosine monophosphate (AMP) and, in animal models, has been shown to increase heart rate in CCB toxicity [21]. However, it has minimal effects on the mean arterial pressure. Glucagon has been effective in treating human cases of CCB toxicity [22-24]. The dosing regimen for glucagon in the treatment of CCB poisoning has not been established in human clinical trials. For the adult patient, an initial 5 mg IV bolus is a reasonable start and may be repeated twice at 10-minute intervals. A glucagon infusion can be started at the total dose at which a response is noted. Nausea and vomiting are common side effects of glucagon administration.

Vasopressors: uptodate recommends norepinephrine. Patients with profound CCB toxicity may require higher doses of vasopressors than those typically used in the treatment of severe sepsis or other causes of shock. In one series, patients with severe diltiazem or verapamil toxicity received vasopressor doses as high as 100 mcg per minute of norepinephrine, 150 mcg per minute of epinephrine, and 100 mcg/kg per minute for dopamine [25]. However, the arrhythmogenic effects of these drugs become more prominent at higher doses; thus, the risk of such high doses, particularly with multiple agents, may outweigh the benefits.

High-dose insulin therapy has positive inotropic effects in patients with CCB toxicity. Its effectiveness and safety have been noted in animal models and case reports [26-29]. Relative hypoglycemia and hypokalemia must be corrected prior to initiating high-dose insulin therapy. For patients with a serum glucose concentration below 8.25 mmol/L, we administer 50 mL of 50 percent dextrose (D50W) IV. For patients with a serum potassium concentration below 3 mmol/L, we administer 20 mEq of potassium IV. We initiate high-dose insulin therapy with a bolus of 1 unit/kg of regular, short-acting insulin given IV. Following this bolus, we begin a continuous infusion at 0.5 units/kg per hour IV and titrate upwards until hypotension is corrected or a maximum dose of 10 units/kg per hour is reached. CCB overdose often causes hyperglycemia that is refractory to high-dose insulin. Therefore, not all patients require supplemental dextrose. If necessary, euglycemia can be maintained by means of a continuous IV infusion of 5 to 10 percent dextrose. A reasonable initial rate for such an infusion is 0.5 to 1 g of dextrose/kg per hour. Titration of the dextrose infusion and additional boluses of 50 percent dextrose are provided based upon serum glucose measurements (finger stick glucose), which are obtained every 15 to 30 minutes until the concentration is stable and every hour thereafter until several hours after therapy is complete.

Lipid emulsions are the fats used in total parenteral nutrition (TPN). Initially used to treat overdoses of local anesthetics such as bupivacaine, IV lipid emulsion (ILE) is being studied as a therapy for poisonings involving a number of lipophilic medications. Studies of ILE therapy are preliminary. Systematic reviews of lipid emulsion therapy for acute poisoning have found the overall quality of studies supporting this treatment to be low or very low, but included human case reports provide some evidence of benefit in patients with toxicity from verapamil, beta blockers, some tricyclic antidepressants, bupivacaine, chlorpromazine, and some antidysrhythmics (eg, flecainide) [33-35]. We suggest consultation with a medical toxicologist or poison control center to determine whether ILE therapy is appropriate.

A transvenous pacemaker can be inserted to assist with electrical conduction. However, pacing does not counteract the negative inotropic effects of these drugs, and successful capture may not correct the patient’s hypotension [49]. If there are no contraindications, an intraaortic balloon pump may help correct intractable hypotension.

Extracorporeal membrane oxygenation — There are several case reports of patients with severe CCB poisoning who survived following treatment with extracorporeal membrane oxygenation (ECMO) [51-53]. We believe that ECMO should be considered a last resort and on a case-by-case basis in conjunction with guidance from a poison control center or medical toxicologist.

Extracorporeal removal — CCBs are highly protein bound; therefore, extracorporeal removal by hemodialysis is not effective.

56
Q

ARDS patient:

a. List two ventilator changes you could make to improve oxygenation
b. List three things you could do to improve mortality
c. List two things that you could do that might improve oxygenation, but that might also cause harm or have no effect on survivial.

A

a)

high PEEP strategy

recruitment maneuver

increase FiO2

APRV

b)

LTVV

fluid restrictive strategy

proning for P:F <150

+/- NMB for P:F <150

c)

recruitment maneuvers

high PEEP strategy

APRV

inhaled pulmonary vasodilators

57
Q

List four confounters to NDD that make ancillary testing necessary

A
  • unable to complete physical exam (i.e. enucleation, amputation etc)
  • supratherapeutic/toxic drug levels (i.e. with sedatives, NMBs)
  • severe lung disease (or history of hypoxic drive etc) that makes physician unsure of the validity of the apnea test
  • possibly early after acute post-resuscitation phase of cardiorespiratory arrest (recommended to wait 24hrs but if can’t then could pursue ancillary test)

NDD CMAJ article:

We recommend that an ancillary test be performed when it is impossible to complete the minimum clinical criteria as defined in Recommendation A.1. At a minimum, 2 particular clinical criteria must be met before ancillary tests are performed:

  • An established etiology capable of causing neurological death in the absence of reversible conditions capable of mimicking neurological death
  • Deep unresponsive coma

We recommend that demonstration of the global absence of intracerebral blood flow be considered as the standard for NDD by ancillary testing.

We recommend that, at the time of assessment for NDD, the following confounding factors preclude the clinical diagnosis:

  • Unresuscitated shock
  • Hypothermia (core temperature < 34°C)
  • Severe metabolic disorders capable of causing a potentially reversible coma
  • Severe metabolic abnormalities, including glucose, electrolytes (including phosphate, calcium and magnesium), inborn errors of metabolism, and liver and renal dysfunction may play a role in clinical presentation. If the primary etiology does not fully explain the clinical picture, and if in the treating physician’s judgement the metabolic abnormality may play a role, it should be corrected.
  • Peripheral nerve or muscle dysfunction or neuromuscular blockade potentially accounting for unresponsiveness
  • Clinically significant drug intoxications (e.g., alcohol, barbiturates, sedatives, hypnotics); however, therapeutic levels or therapeutic dosing of anticonvulsants, sedatives and analgesics do not preclude the diagnosis.
58
Q

Patient with appendectomy. Post op develops hypertension, tachycardia, atrial
fibrillation, and now developing respiratory distress. TSH is <0.04. List four medications you could use to treat.

A

uptodate:

  • A beta blocker to control the symptoms and signs induced by increased adrenergic tone
  • A thionamide to block new hormone synthesis (propyltiouracil or methimazole)
  • An iodine solution to block the release of thyroid hormone
  • Glucocorticoids to reduce T4-to-T3 conversion, promote vasomotor stability, possibly reduce the autoimmune process in Graves’ disease, and possibly treat an associated relative adrenal insufficiency
  • Bile acid sequestrants may also be of benefit in severe cases to decrease enterohepatic recycling of thyroid hormones

For patients with clinical features of thyroid storm or with severe thyrotoxicosis who do not fully meet the criteria for thyroid storm (ie, impending storm), we begin immediate treatment with a beta blocker (propranolol in a dose to achieve adequate control of heart rate, typically 60 to 80 mg orally every four to six hours, with appropriate adjustment for heart rate and blood pressure) and either propylthiouracil (PTU) 200 mg every four hours or methimazole (20 mg orally every four to six hours). PTU is favored over methimazole because of PTU’s effect to decrease T4-to-T3 conversion. One hour after the first dose of thionamide is taken, we administer iodine (saturated solution of potassium iodide [SSKI], 5 drops orally every six hours or Lugol’s solution, 10 drops every eight hours). The administration of iodine should be delayed for at least one hour after thionamide administration to prevent the iodine from being used as substrate for new hormone synthesis in patients with toxic adenoma or toxic multinodular goiter (since the etiology of the thyrotoxicosis is frequently uncertain at the time of admission).

For patients with clinical features of thyroid storm, we also administer glucocorticoids (hydrocortisone, 100 mg intravenously every eight hours). Cholestyramine (4 g orally four times daily) may also be of benefit in severe cases to reduce enterohepatic recycling of thyroid hormones. Supportive therapy and recognition and treatment of any precipitating factors (eg, infection), in addition to specific therapy directed against the thyroid, may be critical to the final outcome. Many patients require substantial amounts of fluid, while others may require diuresis because of congestive heart failure.

Infection needs to be identified and treated, and hyperpyrexia should be aggressively corrected. Acetaminophen should be used instead of aspirin since the latter can increase serum free T4 and T3 concentrations by interfering with their protein binding.

59
Q

Patient with COPD in the ICU for months, tracheostomy in place, is competent to make his own decisions and wants to keep going. We know that ventilator weaning depends on the balance of three factors:
a. What are the three factors of ventilator weaning
b. What two things must be present for you to decide that ventilator weaning is
futile.

A

a) what is this question asking exactly??? only three???

  • systemic disease factors
  • respiratory system mechanical factors (load/capacity balance)
  • iatrogenic factors (inappropriate vent support, upper airway abnormalities, tracheostomies affecting swallowing and aspiration)

b)

  1. treatable conditions that impede weaning have been reversed/improved
  2. documented repeated failure to progress through a weaning protocol, even to reach modest goals (e.g. nocturnal ventilation, invasive/non-invasive)

CHEST 2005 guidelines on prolonged MV

There is no body of evidence to support a set time limit for considering mechanical ventilatory support weaning to be futile, nor when weaning attempts should cease. In the evidence-based weaning guide-lines endorsed by several authoritative bodies, a time of 3 months of mechanical ventilation was extrapolated post hoc from observational studies as being a period when most patients who could be weaned had been weaned.

Relying on a strict timeline to define futility is clearly inappropriate, as many factors enter into the decision to abandon further weaning attempts. On admission to a PMV-focused care venue, the inter-disciplinary team should immediately partner with the patient and family on goal setting and discharge planning. The decision to subsequently consider further weaning futile can then be made based on the following principles: (1) treatable conditions that impede weaning have been reversed/improved as much as possible; and (2) documented repeated failure to progress through a weaning protocol, even to reach modified goals (eg, nocturnal ventilation,invasive or noninvasive). An additional consideration would include a realization that resultant functionality and quality of life from the patient’s standpoint will be unacceptable, even if weaned.

60
Q

Patient with refeeding syndrome:

a. What two changes will you make to TPN?
b. What are three effects of hypophosphatemia?

A

a)

  • decrease total caloric fed to <80% of energy requirements
  • give thiamine before refeeding is instigated

b)

  1. muscle weakness
  2. cardiac arrhythmia
  3. impaired cardiac function
  4. respiratory failure (impaired diaphragm function)
  5. rhabdomyolysis
  6. tremors, paresthesias, delirium, seizures
  7. hemolytic anemia

ASPEN Guidelines:

Management of PN should include attention to rate of advancement of feeding, glycemic control, electrolyte monitoring and repletion (evidence of refeeding), duration of PN, and transition to EN as feasible. Attention to refeeding syndrome is especially important for the patient with risk factors (alcoholism, weight loss, low body mass index [BMI], prolonged periods NPO). Although refeeding syndrome can occur with EN, the risk is higher with initiation of PN. In those patients, advancement of feeding should be slower, taking 3–4 days to reach goal. Use of protocols and nutrition support teams have been shown to decrease PN-associated complications. Permissive underfeeding has also been shown to be a potential shortterm approach to avoid some of these complications.

We suggest that hypocaloric PN dosing (≤20 kcal/kg/d or <80% of estimated energy needs) with adequate protein (≥1.2 g protein/kg/d) be considered in appropriate patients (high risk or severely malnourished) requiring PN, initially over the first week of hospitalization in the ICU.

The NICE guidelines recommend that refeeding is started at no more than 50% of energy requirements in “patients who have eaten little or nothing for more than 5 days.” The rate can then be increased if no refeeding problems are detected on clinical and biochemical monitoring (level D recommendation)…

61
Q

Patient with septic shock, unable to get IV access

a. What are 2 absolute contraindications to intraosseous?
b. What are two relative contraindications?

A

a)

  • bone fracture
  • previous attempt at that site (may be the same as above)
  • An extremity with vascular interruption, either from trauma or cutdown attempt, is not suitable for IO infusion because the fluid or drugs administered into the bone marrow will leak through the open blood vessel.

b)

  • cellulitis, burns, ostemoyelitis
  • osteogenesis imperfecta/osteoporosis
  • R to L intracardiac shunts (ToF, pulmonary atresia) who may be at greater risk for cerebral fat or bone marrow emboli

think skin/surface, bone, and embolic

62
Q

List 4 barriers to proper physician handover.

A

CMPA source

  • communication style (face to face handover allows for interactive questioning)
  • failure to use standardized communication tools
  • setting (if in busy setting, may have frequent interruptions)
  • time constraints (not allowing sufficient time, multi-tasking)
  • missing information
  • lack of training (in giving/getting handover)
63
Q

What three things will allow a researcher to have a smaller sample size in their study?

A

narrow margin of error (aka confidence interval)

high confidence level (the estimated probability that a population estimate lies within a given margin of error)

large effect size (aka treatment effect)

low variability in the outcome variable

anticipated attrition (loss to follow up)

64
Q

Man with pneumonia on FiO2 40%. You increase FiO2 to 100%. What happens to V/Q. 2 reasons\

?should be we calculating shunt fraction to determine if PaO2 will increase or not change?

A

a. Haldane effect – inc pCO2
b. Less hypoxic vasoconstriction

from LHSC

Qs/Qt is a measurement of pulmonary shunt. It describes the percentage of blood that reaches the left side of the heart without picking up oxygen. A normal Qs/Qt is .05….in other words, 5 % of the blood that travels from the lungs to the left side of the heart arrives without picking up oxygen. Qs/Qt represents the pooled average of blood from all regions of the lungs.

A Qs/Qt > .05 indicates that an increased amount of blood is reaching the left side of the heart without picking up new oxygen. This occurs when blood flows past areas of lung where the alveoli have reduced or no ventilation. A V/Q mismatch refers to gas exchange units where blood flow (perfusion) exceeds air flow (ventilation). Areas that are perfused with no ventilation are referred to as physiological or true shunts. When the number of V/Q mismatches or true shunts increase, there is an increased contribution of poorly oxygenated blood to the arterial blood. This results in an increased shunt (Qs/Qt).

Reduced ventilation often occurs as a result of pulmonary edema or secretions. At room air (.21 FiO2), 21% of the air inside the alveoli is oxygen. An increase in the FiO2 doesn’t change the volume of air, rather, it increases the percentage of the air that is oxygen. When V/Q mismatching is present, Qs/Qt will rise. Increased FiO2 levels will often be sufficient to correct the low PaO2 levels that develop. A large increase in the Qs/Qt occurs when the number of true pulmonary shunts increase (unventilated/perfused alveoli). An increase in the FiO2 will have little affect on the PaO2 when true shunting is the cause because air is not reaching the gas exchange area. The only way to increase the PaO2 when true shunting exists is to re-expand the alveoli with mechanical ventilation and PEEP, to increase the surface area for gas exchange.

A Qs/Qt of .10 - .20 indicates an increased shunt that can usually be managed with increased concentrations of inspired oxygen. A shunt of .20 - .30 represents a more serious oxygenation problem. A shunt of .31 represents a very significant gas exchange deficit, where increased FiO2 levels are generally insufficient to correct the problem. Mechanical ventilation with increased PEEP will be needed to improve the low arterial oxygen level (by reinflating areas of reduced ventilation).

deranged physiology

65
Q

Man thrown off horse, landed prone, horse landed on back. Presents to ED/Trauma with sensation to angle of Louis, motor – strongest to D1, weak to D5 both hands, no movement in lower extremities. HR 50, BP 80/60, Sat 98 on RA, EtCO2 52. Assume only injury is SCI (i.e. no hemorrhage)

a. What dermatome is his level.
b. Pathophysiology for hypotension.
c. 4 interventions to consider immediate management.

A

?what does D1 and D5 refer to?

a) angle of Louis seems to refer to C4 dermatome
b) neurogenic shock, see full description below, occurs with disruption/damage to sympathetic nerve fibers typically with traumatic SCI above T6 which results in a state of unopposed parasympathetic (i.e. vagal) stimulation leading to vasoplegia, hypotension, bradycardia, temperature dysregulation.

c)

  1. IVF
  2. atropine +/- external pacing
  3. vasopressors to target MAP 85-90
  4. C-spine stabilization +/- surgery to stabilize C spine (which may help with neurogenic shock)

Grade — Definition

A — Complete. No sensory or motor function is preserved in the sacral segments S4-S5

B — Incomplete. Sensory but not motor function is preserved below the neurological level and includes the sacral segments S4-S5

C — Incomplete. Motor function is preserved below the neurological level, and more then half of key muscles below the neurological level have a muscle grade less then 3 (Grades 0-2).

D — Incomplete. Motor function is preserved below the neurological level, and at least half of key muscles below the neurological level have a muscle grade greater than or equal to 3.

E — Normal. Sensory and motor functions are normal.

Neurogenic shock refers to hypotension, usually with bradycardia, attributed to interruption of autonomic pathways in the spinal cord causing decreased vascular resistance. Patients with TSCI may also suffer from hemodynamic shock related to blood loss and other complications. An adequate blood pressure is believed to be critical in maintaining adequate perfusion to the injured spinal cord and thereby limiting secondary ischemic injury. Albeit with few empiric supporting data, guidelines currently recommend maintaining mean arterial pressures of at least 85 to 90 mmHg and using intravenous (IV) fluids, transfusion, and pharmacologic vasopressors as needed.

While controlled hypotension may be appropriate for many trauma patients, those with SCI, like those with traumatic brain injury, are at greater risk for complications from hypotension. Thus, a higher blood pressure is needed to ensure adequate perfusion of the spinal cord. Guidelines from 2013 recommend maintaining a MAP of at least 85 to 90 mmHg for patients with SCI [40], and subsequent studies support this goal. While there is no clearly defined combination of blood products and vasopressor treatment for these patients, norepinephrine is recommended over dopamine or phenylephrine by some authorities when vasopressors are required for patients with SCI, and we concur with this approach.

Neurogenic shock is a devastating consequence of spinal cord injury (SCI), also known as vasogenic shock. Injury to the spinal cord results in sudden loss of sympathetic tone, which leads to the autonomic instability that is manifested in hypotension, bradyarrhythmia, and temperature dysregulation. Spinal cord injury is not to be confused with spinal shock, which is a reversible reduction in sensory and motor function following spinal cord injury. Neurogenic shock is associated with cervical and high thoracic spine injury.

Neurogenic shock is defined as the injury to the spinal cord with associated autonomic dysregulation. This dysregulation is due to a loss of sympathetic tone and unopposed parasympathetic response. Neurogenic shock is most commonly a consequence of traumatic spinal cord injuries.

Neurogenic shock is the clinical state manifested from primary and secondary spinal cord injury. Hemodynamic changes are seen with an injury to the spinal cord above the level of T6. The descending sympathetic tracts are disrupted most commonly from associated fracture or dislocation of vertebrae in the cervical or upper thoracic spine. Primary spinal cord injury occurs within minutes of initial insult. Primary injury is direct damage to the axons and neural membranes in the intermediolateral nucleus, lateral grey mater, and anterior root that lead to disrupted sympathetic tone. Secondary spinal cord injury occurs hours to days after the initial insult. Secondary injury is a result of vascular insult, electrolyte shifts, and edema that lead to progressive central hemorrhagic necrosis of grey matter at the injury site. At a cellular level, there is excitotoxicity from NMDA accumulation, improper homeostasis of electrolytes, mitochondrial injury, and reperfusion injury which all lead to controlled and uncontrolled apoptosis. Neurogenic shock is a combination of both primary and secondary injury that lead to loss of sympathetic tone and thus unopposed parasympathetic response driven by the Vagus nerve. Consequently, patients suffer from instability in blood pressure, heart rate, and temperature regulation.

66
Q

4 clinical or investigations that portend poor prognosis following cardiac arrest

A

AHA Guidelines 2015 Recommendations (of note image is from European guidelines)

TIMING

The earliest time for prognostication using clinical examination in patients treated with TTM, where sedation or paralysis could be a confounder, may be 72 hours after return to normo- thermia (Class IIb, LOE C-EO).

We recommend the earliest time to prognosticate a poor neurologic outcome using clinical examination in patients not treated with TTM is 72 hours after cardiac arrest (Class I, LOE B-NR). This time until prognostication can be even longer than 72 hours after cardiac arrest if the residual effect of sedation or paralysis confounds the clinical examination (Class IIa, LOE C-LD).

Operationally, the timing for prognostication is typically 4.5 to 5 days after ROSC for patients treated with TTM. This approach minimizes the possibility of obtaining false-positive results (ie, inaccurately suggesting a poor outcome) because of drug-induced depression of neurologic function. In making this recommendation, it is recognized that in some instances, withdrawal of life support may occur appropriately before 72 hours because of underlying terminal disease, brain herniation, or other clearly nonsurvivable situations.

CLINICAL EXAM

In comatose patients who are not treated with TTM, the absence of pupillary reflex to light at 72 hours or more after cardiac arrest is a reasonable exam finding with which to predict poor neurologic outcome (FPR, 0%; 95% CI, 0%–8%; Class IIa, LOE B-NR).

In comatose patients who are treated with TTM, the absence of pupillary reflex to light at 72 hours or more after cardiac arrest is useful to predict poor neurologic outcome (FPR, 1%; 95% CI, 0%–3%; Class I, LOE B-NR).

We recommend that, given their unacceptable FPRs, the findings of either absent motor movements or extensor posturing should not be used alone for predicting a poor neurologic outcome (FPR, 10%; 95% CI, 7%–15% to FPR, 15%; 95% CI, 5%–31%; Class III: Harm, LOE B-NR). The motor examination may be a reasonable means to identify the population who need further prognostic testing to predict poor outcome (Class IIb, LOE B-NR).

We recommend that the presence of myoclonus, which is distinct from status myoclonus, should not be used to predict poor neurologic outcomes because of the high FPR (FPR, 5%; 95% CI, 3%–8% to FPR, 11%; 95% CI, 3%–26%; Class III: Harm, LOE B-NR).

In combination with other diagnostic tests at 72 or more hours after cardiac arrest, the presence of status myoclonus during the first 72 to 120 hours after cardiac arrest is a reasonable finding to help predict poor neurologic outcomes (FPR, 0%; 95% CI, 0%–4%; Class IIa, LOE B-NR).

EEG

In comatose post–cardiac arrest patients who are treated with TTM, it may be reasonable to consider persistent absence of EEG reactivity to external stimuli at 72 hours after cardiac arrest, and persistent burst suppression on EEG after rewarming, to predict a poor outcome (FPR, 0%; 95% CI, 0%–3%; Class IIb, LOE B-NR).

Intractable and persistent (more than 72 hours) status epilepticus in the absence of EEG reactivity to external stimuli may be reasonable to predict poor outcome (Class IIb, LOE B-NR).

In comatose post–cardiac arrest patients who are not treated with TTM, it may be reasonable to consider the presence of burst suppression on EEG at 72 hours or more after cardiac arrest, in combination with other predictors, to predict a poor neurologic outcome (FPR, 0%; 95% CI, 0%–11%; Class IIb, LOE B-NR).

EVOKED POTENTIALS

In patients who are comatose after resuscitation from cardiac arrest regardless of treatment with TTM, it is reasonable to consider bilateral absence of the N20 SSEP wave 24 to 72 hours after cardiac arrest or after rewarming a predictor of poor outcome (FPR, 1%; 95% CI, 0%–3%; Class IIa, LOE B-NR).

SSEP recording requires appropriate skills and experience, and utmost care should be taken to avoid electrical interference from muscle artifacts or from the intensive care unit environment. However, sedative drugs or temperature manipulation affect SSEPs less than they affect the EEG or clinical examination.

IMAGING

In patients who are comatose after resuscitation from cardiac arrest and not treated with TTM, it may be reasonable to use the presence of a marked reduction of the GWR on brain CT obtained within 2 hours after cardiac arrest to predict poor outcome (Class IIb, LOE B-NR).

It may be reasonable to consider extensive restriction of diffusion on brain MRI at 2 to 6 days after cardiac arrest in combination with other established predictors to predict a poor neurologic outcome (Class IIb, LOE B-NR).

Acquisition and interpretation of imaging studies have not been fully standardized and are subject to interobserver variability. In addition, the recommendations for brain imaging studies for prognostication are made with the assumption that images are performed in centers with expertise in this area.

BIOMARKERS

Given the possibility of high FPRs, blood levels of NSE and S-100B should not be used alone to predict a poor neurologic outcome (Class III: Harm, LOE C-LD).

When performed with other prognostic tests at 72 hours or more after cardiac arrest, it may be reasonable to consider high serum values of NSE at 48 to 72 hours after cardiac arrest to support the prognosis of a poor neurologic outcome (Class IIb, LOE B-NR), especially if repeated sampling reveals persistently high values (Class IIb, LOE C-LD).

Laboratory standards for NSE and S-100B measurement vary between centers, making comparison of absolute values difficult.

67
Q

5 risk factors for invasive candidiasis in a non-neutropenic patient

A
  • TPN
  • CVC
  • broad spectrum Abx
  • high APACHE scores
  • acute renal failure (especially if requiring hemodialysis)
  • prior surgery (especially abdominal surgery)
  • I think ICU admission is in and of itself a risk factor too
  • burn injury

immunocompromised pts

  • hematologic malignancies
  • recipients of solid organ or hematopoietic cell transplants
  • those given chemotherapeutic agents, especially those associated with extensive GI mucosal damage

from another article (good way to think about it):

Risk factors for IC may be assigned into two groups: host-related factors and health-care-associated factors including catheter use, total parenteral nutrition, surgical interventions, and the use of antimicrobial drugs. The leading host-related factors are immunosuppressive diseases, neutropenia, age, and a deteriorating clinical condition due to underlying diseases.2,28,29,31 The most common health-care-associated risks are long hospital or ICU stay.1 Risk factors associated with long ICU stay include invasive interventions and colonization. Candida colonization is a risk factor, the importance of which has been realized in recent years. According to various studies, this risk factor for IC development is more related to the presence or absence of colonization than the number of regions colonized.68,69 However, the detection of colonization in any part of the body is only a risk factor, not a disease, and treatment should not be started. Regardless, the lack of Candida colonization is a strong indicator in favor of excluding IC diagnosis.

68
Q

Anaphylactic Shock

a. Most important drug to give (be specific)
b. 3 other pharmacologic things to give

A

a) epinephrine IV or IM (IM injection is also preferred over subcutaneous or intravenous (IV) bolus because it is faster in many situations and safer (ie, lower risk of cardiovascular complications, such as severe hypertension and ventricular arrhythmias).

  • IM dosing – For situations where an exact dose can be drawn up and administered, the recommended dose of epinephrine for patients of any age is 0.01 mg/kg (maximum dose of 0.5 mg - I would just give that to all adults) per single dose, injected IM into the mid-outer thigh (vastus lateralis muscle). The dose should be drawn up using a 1 mL syringe using the 1 mg/mL formulation of epinephrine
  • Patients who do not respond to several IM injections of epinephrine and aggressive fluid resuscitation may not be adequately perfusing muscle tissues, as most commonly occurs in individuals presenting with profound hypotension or symptoms and signs suggestive of impending shock (dizziness, incontinence of urine and/or stool).

Such patients should receive epinephrine by slow intravenous (IV) infusion, with the rate titrated according to response and the presence of continuous electronic hemodynamic monitoring. Start the IV epinephrine infusion at 0.1 mcg/kg/minute and increase it every two to three minutes by 0.05 mcg/kg/minute until BP and perfusion improve.

  • Glucagon for patients taking beta-blockers — Beta-blocker therapy, alone or in combination with angiotensin-converting enzyme inhibitors, has been associated with more severe anaphylaxis [70]. However, in a retrospective study of 789 patients presenting to the emergency department, patients taking beta-blockers were no more likely to need epinephrine or to need additional doses of epinephrine than patients not taking beta-blockers, so the clinical impact may be relatively small [71]. Nonetheless, although epinephrine should still be used at first-line therapy, even in patients taking beta-blockers, if a patient taking beta-blockers appears to be refractory to epinephrine, glucagon can be administered because it has inotropic and chronotropic effects that are not mediated through beta-receptors

b)

  • glucocorticoids for rebound
  • H1 blocker diphenhydramine
  • H2 blocker ranitidine
  • salbutamol (for epinephrine-resistant bronchospasm)
  • could consider vasopressin if no longer responsive to epi (targest different receptors)
  • hail mary options: methylene blue, ECMO
69
Q

Stem with cardiogenic shock

a. vasopressor proven to have more adverse effects
b. Best vasopressor to give the most chronotropy

A

a)Initially I would have said dobutamine but technically I wouldn’t classify that as a vasopressor so I suppose epi becuase of risk of arrhythmia???

b)

I don’t think it’s possible to answer this question. The table in the AHA statement lists epi, dob, and isoproterenol as all having ++++ B1 activity and it does not give specific recommendations for bradycardia induced cardiogenic shock…

From AHA Statement on CS

We suggest that when an early invasive approach cannot be completed in a timely fashion, fibrinolysis can be considered in CS associated with STEMI. The decision to administer fibrinolysis should be individualized on the basis of perceived reperfusion benefit, bleeding risks, and the anticipated time delay to angiography.

We support guidelines that recommend an early invasive strategy with appropriate revascularization for all suitable patients with suspected ACS-associated CS, including patients with uncertain neurological status or those who have received prior fibrinolysis, regardless of the time delay from MI onset.

In summary, evidence continues to support the early revascularization of patients with CS after ACS, with either PCI or CABG used as indicated. Until the results of CULPRIT-SHOCK are available, revascularization of both the culprit and hemodynamically significant non-culprit stenoses is reasonable. We support the preferential use of radial arterial access for angiography and PCI when feasible.

We suggest that all patients with CS without serious bleeding complications be continued on dual antiplatelet therapy without interruption after PCI. In situations when oral agents cannot be administered or there are concerns about absorption, the use of an intravenous glycoprotein IIb/IIIa inhibitor or the recently available intravenous P2Y 12 inhibitor cangrelor can be considered. No high-quality data are available to support the efficacy or safety of glycoprotein IIb/IIIa inhibitors in patients with MCS.

We suggest the use of PACs in cases of diagnostic or CS management uncertainty or in patients with moderate to severe CS who are unresponsive to initial therapy.

Hemodynamic monitoring should complement (and not replace) other markers of end-organ perfusion in CS. The optimal MAP likely differs from patient to patient, and the risks of hypoperfusion with lower MAPs must be balanced (and individualized) with the potentially deleterious impact of vasoactive agents on myocardial oxygen demand, ischemia, and arrhythmia associated with higher MAP targets. We suggest that clinicians assess the adequacy of end-organ and tissue perfusion in response to individualized targets by integrating serial markers of systemic perfusion, including (but not limited to) arterial lactate, mixed or central venous oxygen saturations, urine output, creatinine, liver function tests, mental status, temperature, and other invasive hemodynamic variables.

We support guideline recommendations for the management of patients with STEMI that suggest avoidance of β-blockers in patients with signs of HF or low-output states and the avoidance of RAAS antagonists in patients with hypotension. It may be reasonable to initiate β-blockers when the patient is euvolemic and off inotropes and vasopressors for at least 24 hours. RAAS inhibitor therapy initiation can be considered when the patient has been off vasopressors for 24 hours, provided that the patient’s renal function has returned nearly to baseline levels and the risk of RAAS-associated hyperkalemia or hypotension is low. RAAS inhibitors may be started in patients with pulmonary edema and in conjunction with an inodilator.

We suggest that it is reasonable to administer statin in patients with MI-associated CS.

Norepinephrine is associated with fewer arrhythmias and may be the vasopressor of choice in many patients with CS; however, in light of the aforementioned major study limitations, the optimal first-line vasoactive medication in CS remains unclear.

70
Q

3 reasons why a random cortisol doesn’t reflect adrenal function in critically ill patient

A

Total serum cortisol levels vary widely in patients with septic shock [5,18-23]. There is no robust relationship between serum cortisol levels and mortality in patients with septic shock [19,21,24-31]. One prospective study of 101 patients with sepsis reported that the best predictor of adrenal insufficiency (as measured by an overnight ACTH stimulation test) was baseline random cortisol level of ≤10 microg/dL or a change of cortisol of <9 microg/dL.

  1. increase in the free to bound ratio and a decrease in serum cortisol-binding capacity attributable to changes in CBG and albumin
  2. ???cortisol ranges vary widely in critical illness??? total guess
  3. altered cortisol metabolism in critical illness